You are on page 1of 68

Pediatrics Examination and Board

Review (Oct 3,
2016)_(0071847685)_(McGraw Hill) 1st
Edition Peterson
Visit to download the full and correct content document:
https://ebookmass.com/product/pediatrics-examination-and-board-review-oct-3-2016_
0071847685_mcgraw-hill-1st-edition-peterson/
M c G R A W- H I L L E D U C A T I O N
SPECIALTY BOARD REVIEW

Pediatrics
Examination
and Board Review

• 1,500+ board-style questions


with detailed answer discussions

•780+ progressive clinical cases


•Comprehensive final exam

ANDREW PETERSON
Gra w KELLY WOOD
Hill
Education
McGraw-Hill Education Specialty Board Review

PEDIATRICS
EXAMINATION
AND BOARD

REVIEW
McGraw-Hill Education Specialty Board Review
PEDIATRICS
EXAMINATI N
fir-4"...”-

Edited by

‘ Andrew R. Peterso Kelly E. Wood, MD


Clinical Associate Professor Clinical Assistant Professor
Stead Family Department of Pediatrics Stead Family Department of Pediatrics
Carver College of Medicine Carver College of Medicine
University of Iowa University of Iowa
Iowa City, Iowa Iowa City, Iowa

New York / Chicago / San Francisco / Athens / London / Madrid / Mexico City ‘
Milan / New Delhi / Singapore / Sydney /Toronto
Copyright © 2017 by McGraw-Hill Education. All rights reserved. Except as permitted under the United States Copyright Act of 1976,
no part of this publication may be reproduced or distributed in any form or by any means, or stored in a database or retrieval system,
without the prior written permission of the publisher.

ISBN: 978-0-07-184769-8

MHID: 0-07-184769-3.

The material in this eBook also appears in the print version of this title: ISBN: 978-0-07-184768-1,
MHID: 0-07-184768-5.

eBook conversion by codeMantra


Version 1.0

All trademarks are trademarks of their respective owners. Rather than put a trademark symbol after every occurrence of a trademarked
name, we use names in an editorial fashion only, and to the benefit of the trademark owner, with no intention of infringement of the
trademark. Where such designations appear in this book, they have been printed with initial caps.

McGraw-Hill Education eBooks are available at special quantity discounts to use as premiums and sales promotions or for use in corpo­
rate training programs. To contact a representative, please visit the Contact Us page at www.mhprofessional.com.

Notice
Medicine is an ever-changing science. As new research and clinical experience broaden our knowledge, changes in treatment and drug
therapy are required. The authors and the publisher of this work have checked with sources believed to be reliable in their efforts to
provide information that is complete and generally in accord with the standards accepted at the time of publication. However, in view
of the possibility of human error or changes in medical sciences, neither the authors nor the publisher nor any other party who has been
involved in the preparation or publication of this work warrants that the information contained herein is in every respect accurate or com­
plete, and they disclaim all responsibility for any errors or omissions or for the results obtained from use of the information contained in
this work. Readers are encouraged to confirm the information contained herein with other sources. For example and in particular, readers
are advised to check the product information sheet included in the package of each drug they plan to administer to be certain that the
information contained in this work is accurate and that changes have not been made in the recommended dose or in the contraindications
for administration. This recommendation is of particular importance in connection with new or infrequently used drugs.

TERMS OF USE

This is a copyrighted work and McGraw-Hill Education and its licensors reserve all rights in and to the work. Use of this work is subject
to these terms. Except as permitted under the Copyright Act of 1976 and the right to store and retrieve one copy of the work, you may
not decompile, disassemble, reverse engineer, reproduce, modify, create derivative works based upon, transmit, distribute, disseminate,
sell, publish or sublicense the work or any part of it without McGraw-Hill Education's prior consent. You may use the work for your
own noncommercial and personal use; any other use of the work is strictly prohibited. Your right to use the work may be terminated if
you fail to comply with these terms.

THE WORK IS PROVIDED "AS IS." McGRAW-HILL EDUCATION AND ITS LICENSORS MAKE NO GUARANTEES OR WAR­
RANTIES AS TO THE ACCURACY, ADEQUACY OR COMPLETENESS OF OR RESULTS TO BE OBTAINED FROM USING
THE WORK, INCLUDING ANY INFORMATION THAT CAN BE ACCESSED THROUGH THE WORK VIA HY PERLINK OR
OTHERWISE, AND EXPRESSLY DISCLAIM ANY WARRANTY, EXPRESS OR IMPLIED, INCLUDING BUT NOT LIMITED
TO IMPLIED WARRANTIES OF MERCHANTABILITY OR FITNESS FOR A PARTICULAR PURPOSE. McGraw-Hill Education
and its licensors do not warrant or guarantee that the functions contained in the work will meet your requirements or that its opera­
tion will be uninterrupted or error free. Neither McGraw-Hill Education nor its licensors shall be liable to you or anyone else for any
inaccuracy, error or omission, regardless of cause, in the work or for any damages resulting therefrom. McGraw-Hill Education has no
responsibility for the content of any information accessed through the work. Under no circumstances shall McGraw-Hill Education and/
or its licensors be liable for any indirect, incidental, special, punitive, consequential or similar damages that result from the use of or
inability to use the work, even if any of them has been advised of the possibility of such damages. This limitation of liability shall apply
to any claim or cause whatsoever whether such claim or cause arises in contract, tort or otherwise.
For VAC, RAP, MEP, MKP, KJT, and W. Thanks.
Andrew R. Peterson

To my family and friends. You made his happen.


Kelly E. Wood
Contents

Contributors 0 0 0 0 0 0 0 0 0 0 0 0 0 0 0 0 0 0 0 0 0 0 0 0 0 0 0 0 0 0 0 0 0 0 0 0 0 0 0 0 0 0 0 0 ix 20 I nfecti o u s Di sease 42 1


Preface 0 0 0 0 0 0 0 0 0 0 0 0 0 0 0 0 0 0 0 0 0 0 0 0 0 0 0 0 0 0 0 0 0 0 0 0 0 0 0 0 0 0 0 0 0 0 0 0 0 xiii 21 Meta b o l i c Di sorders 459
Ado l escent Med ic i n e a n d Gynecology 0 0 0 0 0 0 0 0 0 0 0
22 M u scu loske l etal Di sorders 48 1
2 A l l e rg i c a n d I m m u nologic Di sorders 25 23 N e u rologic Di sorders 51 1
3 Behaviora l and Mental Health Issues 51 24 N utrition 539
4 B l ood a n d Neoplastic Di sorders 0 0 0 0 0 0 0 0 0 0 0 0 0 0 0 0 0 73 25 Patient Safety a n d Q u a l ity I m p rovement 561
5 Ca rd i o l ogy 0 0 0 0 0 0 0 0 0 0 0 0 0 0 0 0 0 0 0 0 0 0 0 0 0 0 0 0 0 0 0 0 0 0 0 0 1 09 26 P h a rmacology: Pa i n Ma nagement
6 Cog nition, La n g u age, a n d Lea r n i n g a n d Sedation 0 0 0 0 0 0 0 0 0 0 0 0 0 0 0 0 0 0 0 0 0 0 0 0 0 0 0 0 0 0 0 0 0 0 573
Disabi l ities 0 0 0 0 0 0 0 0 0 0 0 0 0 0 0 0 0 0 0 0 0 0 0 0 0 0 0 0 0 0 0 0 0 0 0 0 1 31 27 Poi so n i n g a n d Envi ro n menta l
7 Co l l a g e n Va sc u l a r a n d Other M u ltisyste m Expo s u re t o Hazard o u s S u bsta nces 595
Di sorders 1 51 28 Preve ntative Ped iatrics 625
8 Critica l Care
0 0 0 0 0 0 0 0 0 0 0 0 0 0 0 0 0 0 0 0 0 0 0 0

1 65 29 Psych osoc i a l Issues a n d C h i l d Abuse 643


9 Ea r, Nose, a n d Th roat Di sorders
0 0 0 0 0 0 0 0 0 0 0

1 79 30 Research a n d Statistics 66 1
10 Emerg e ncy Ca re 203 31 Ren a l a n d U rologic Disord e rs 0 0 0 0 0 0 0 0 0 0 0 0 0 0 0 0 0 0 669
11 Endocrine Di sorders 21 9 32 Res pi ratory Di sorders 0 0 0 0 0 0 0 0 0 0 0 0 0 0 0 0 0 0 0 0 0 0 0 0 0 0 697
12 Eth i cs 241 33 Skin Di sorders 727
13 Eye Di sorders 0 0 0 0 0 0 0 0 0 0 0 0 0 0 0 0 0 0 0 0 0 0 0 0 0 0 0 0 0 0 0 0 0 0 255 34 Sports Med ic i n e and Physical Fitness 759
14 Fetus a n d Newborn
0 0 0 0 0 0 0 0 0 0 0

0 0 0 0 0 0 0 0 0 0 0 0 0 0 0 0 0 0 0 0 0 0 0 0 0 0 0 275 35 S u bsta nce Abuse 779


15 Fluid a n d El ectro lyte Meta bol i s m 0 0 0 0 0 0 0 0 0 0 0 0 0 0 299 Fi n a l Exa m 79 1
16 Ped iatric Gastroenterology 0 0 0 0 0 0 0 0 0 0 0 0 0 0 0 0 0 0 0 0 323 An swe r Key 807
1 7 Genetics a n d Dysmorphology 0 0 0 0 0 0 0 0 0 0 0 0 0 0 0 0 0 361
1 8 Gen ital Syste m Di sorders 377 Index 809
19 G rowth and Development 403

VII
Contributors

Dina Al-Zubeidi, MD Paula Cody, MD, MPH


Clinical Assistant Professor of Pediatrics Assistant Professor
Division of Gastroenterology Department of Pediatrics
Stead Family Department of Pediatrics, University of Wisconsin School of Medicine
Carver College of Medicine and Public Health
University oflowa Madison, Wisconsin [ 1 ]
Iowa City, Iowa [ 1 6]
Cassandra J . Collins, BSW, LISW
William Aughenbaugh, MD Clinical Social Worker
Associate Professor and Program Director Department of Social Service
Department of Dermatology University oflowa Hospitals & Clinics
Vice Chair of Education in the Department of Dermatology Iowa City, Iowa [29]
and Director of Specialty Clinical Medical Education and
Residency Preparation Amy L. Conrad, PhD
University of Wisconsin Assistant Professor
Madison, Wisconsin [33] The Stead Family Department of Pediatrics
University oflowa Children's Hospital
LaTisha L. Bader, PhD, LP, LAC,CC-AASP Iowa City, Iowa [ 6]
Center for Dependence, Addiction and Rehabilitation
(CeDAR) Linda J. Cooper-Brown, PhD
University of Colorado Hospital Clinical Associate Professor
Aurora, Colorado [35] Stead Family Department of Pediatrics,
Division of Pediatric Psychology
Rebecca Benson, MD, PhD The University of Iowa
Medical Director, Pediatric Pain and Palliative Care Program Iowa City, Iowa [3]
Medical Director for Clinical Ethics and Director,
Ethics Consult Service Vanessa A. Curtis, MD
Stead Family Department of Pediatrics Clinical Assistant Professor
University oflowa Children's Hospital, Department of Pediatrics
University oflowa Hospitals and Clinics Division of Endocrinology and Diabetes
Iowa City, Iowa [ 1 2] University of Iowa Carver College of Medicine
Iowa City, Iowa [ 1 1 ]
James D. Burkhalter, LISW
Director of DBT Programming Anthony J. Fischer, MD, PhD
Social Work Specialist, Department of Psychiatry Assistant Professor
University oflowa Hospitals and Clinics Department of Pediatrics
Iowa City, Iowa [29] Division of Allergy, Pulmonology, and Immunology
University oflowa Children's Hospital
Gayathri Chelvakumar, MD, MPH Iowa City, Iowa [32]
Nationwide Children's Hospital
Section of Adolescent Medicine
Columbus, Ohio [ 1 ]

IX
X Contributors

Chris Hogrefe, MD, FACEP Elizabeth H. Mack, MD, MS


Assistant Professor Associate Professor of Pediatrics
Department of Medicine-Sports Medicine Division of Pediatric Critical Care
Department of Emergency Medicine Medical University of South Carolina
Department of Orthopaedic Surgery-Sports Medicine Charleston, South Carolina [25]
Northwestern Medicine
Northwestern University Feinberg School of Medicine Jessie Marks, MD, FAAP
Chicago, Illinois [27] Clinical Assistant Professor
University of Iowa Carver College of Medicine
Sandy D. Hong, MS, MD Stead Family Department of Pediatrics,
Assistant Clinical Professor University oflowa Children's Hospital
Division of Rheumatology Iowa City, Iowa [ 14]
Department of Pediatrics
University oflowa Children's Hospital Ross Mathiasen, MD
Iowa City, Iowa [7] Department of Emergency Medicine
Department of Family Medicine
Erin Howe, MD Institute for Orthopaedics, Sports Medicine, and Rehabilitation
University oflowa Stead Family The University of Iowa Carver College of Medicine
Department of Pediatrics Iowa City, Iowa [35]
Iowa City, Iowa [ 1 9]
Satsuki Matsumoto, MD
Jennifer G. Jetton, MD Associate of Pediatrics
Clinical Assistant Professor Department of Pediatrics, Division of Neurology and
Division of Pediatric Nephrology, Dialysis and Transplantation Developmental and Behavioral Pediatrics
Stead Family Department of Pediatrics Roy J. and Lucille A. Carver College of Medicine
University oflowa Children's Hospital University oflowa
Iowa City, Iowa [3 1 ] Iowa City, Iowa [23]

Kathleen Kieran, MD, MS Jennifer McWilliams, MD


Associate Professor of Urology Child and Adolescent Psychiatrist
Department of Urology Department of Behavioral Health
University of Washington/Seattle Children's Hospital Children's Hospital and Medical Center
Seattle, Washington [ 1 8] Omaha, Nebraska [3]

Todd Kopelman, PhD, BCBA Gary Milavetz, Pharm D, FCCP, FAPhA


Clinical Assistant Professor Associate Professor and Division Head
Department of Psychiatry The University of Iowa College of Pharmacy
University oflowa Hospitals and Clinics Department of Pharmacy Practice and Science
Iowa City, Iowa [ 6] Division of Applied Clinical Sciences
Iowa City, Iowa [26]
Kathy Lee-Son, MD, MHSc
Clinical Assistant Professor Sarah L. Miller, MD
Pediatric Nephrology Department Clinical Assistant Professor
Stead Family University oflowa Children's Hospital Department of Emergency Medicine
Iowa City, Iowa [3 1 ] University oflowa Hospitals and Clinics
Iowa City, Iowa [ 1 0]
Ashley Loomis, MD
Assistant Professor, Pediatric Critical Care Ashley A. Miller, MD, FAAP
University of Minnesota Pediatrician
Minneapolis, Minnesota [8] Pediatrics
Geisel School of Medicine at Dartmouth
Rebecca L. Lozman-Oxman, DNP, APRN, Hanover, New Hampshire [28]
MSN, BSN, MPH
Pediatric Nurse Practitioner Lisa K. Muchard, MD
Pediatrics, New London Hospital/Newport Assistant Clinical Professor
Health Center Department of Dermatology
New London, New Hampshire [28] University of Wisconsin
Madison, Wisconsin [33]
Contributors XI

Blaise Nemeth, MD, MS Judith Regine Sabah, MD, PhD, MBA


Associate Professor (CHS) Ophthalmologist -Comprehensive
Pediatric Orthopedics, American Family and Pediatric/Adult Strabismus
Children's Hospital Operative Care, Eugene VA Healthcare System
Department of Orthopedics and Rehabilitation Eugene, Oregon [ 1 3 ]
University of Wisconsin School of
Medicine and Public Health Melanie A. Schmitt, MD
Madison, Wisconsin [22] Assistant Professor of Pediatric Ophthalmology
and Director of Ophthalmic Genetics
Benton Ng, MD Department of Ophthalmology and Visual Sciences
Pediatric Cardiologist University of Wisconsin-Madison
Pediatrics, All Children's Hospital Madison, Wisconsin [ 1 3 ]
St. Petersburg, Florida [5]
Laura Steinauer, Pharm D Candidate
Erin A. Osterholm, MD Student Pharmacist
Assistant Professor of Pediatrics The University of Iowa College of Pharmacy
Department of Pediatrics, Division of Neonatology Iowa City, Iowa [26]
University of Minnesota
Minneapolis, Minnesota [ 14] Natalie Stork, MD
Assistant Professor
Niyati Patel, MD University of Missouri-Kansas City School of Medicine
Assistant Professor, Pediatric Critical Care Department of Orthopedic Surgery
University of Minnesota and Department of Pediatrics
Minneapolis, Minnesota [8] The Children's Mercy Hospital, Division of Orthopedics
Section of Sports Medicine
Andrew R. Peterson, MD, MSPH Kansas City, Missouri [22]
Clinical Associate Professor
Stead Family Department of Pediatrics, Alex Thomas, MD
Carver College of Medicine Allergist/Immunologist
University of lowa Internal Medicine/Pediatrics
Iowa City, Iowa [30, 34] Presence Sts. Mary and Elizabeth Medical Center,
Advocate Children's Hospital
Catherina Pinnaro, MD Chicago, Illinois [2]
Pediatrics Resident, Department of Pediatrics
University of lowa Amy 0. Thomas, MD
Iowa City, Iowa [ 1 2] Allergist/Immunologist
Allergy and Immunology Department
Nathan Price, MD Dreyer Medical Clinic- Advocate Hospital System
Clinical Assistant Professor Aurora, Illinois [2]
Pediatric Infectious Diseases
Stead Family Department of Pediatrics Elizabeth C. Utterson, MD
Iowa City, Iowa [20] Assistant Professor of Pediatrics
Division of Pediatric Gastroenterology,
Gregory M. Rice, MD Hepatology and Nutrition
Associate Professor of Pediatrics, Division of Genetics Washington University
and Metabolism St. Louis, Missouri [ 1 6]
Co- Director, WSLH Biochemical Genetics Laboratory;
Director, Medical Genetics Residency Program Jeffrey Robert Van Blarcom, MD
University of Wisconsin School of Medicine and Public Health Assistant Professor
Madison, Wisconsin [ 1 7] Department of Pediatrics, Division of Inpatient Medicine
University of Utah
Eric T. Rush, MD, FAAP, FACMG Salt Lake City, Utah [ 1 5, 26]
Departments of Pediatrics and Internal Medicine
University of Nebraska Medical Center Susan S. Vos, PharmD, BCPS, FAPhA
and Children's Hospital and Medical Center Clinical Associate Professor
Omaha, Nebraska [2 1 ] The University oflowa College of Pharmacy
Department of Pharmacy Practice and Science
Division of Applied Clinical Sciences
Iowa City, Iowa [26]
XII Contri butors

Tammy L. Wilgenbusch, PhD Leah Zhorne, MD


Clinical Assistant Professor Clinical Assistant Professor
Stead Family Department of Pediatrics, Department of Pediatrics, Division of Neurology and
Division of Psychology Developmental and Behavioral Pediatrics
University oflowa Children's Hospital Roy J. and Lucille A. Carver College of Medicine
Iowa City, Iowa [ 6] University oflowa
Iowa City, Iowa [23]
Adam D. Wolfe, MD, PhD
Assistant Professor of Pediatric Hematology-Oncology Derek Zhorne, MD
Baylor College of Medicine Clinical Assistant Professor of Pediatrics
Children's Hospital of San Antonio Division of General Pediatrics and Adolescent Medicine
San Antonio, Texas [4] Stead Family Department of Pediatrics
Iowa City, Iowa [9]
Kelly E. Wood, MD
Clinical Assistant Professor
Stead Family Department of Pediatrics
Carver College of Medicine
University oflowa
Iowa City, Iowa [24, 29]
Preface

Welcome to the Pediatrics Examination and Board Review Pediatricians trying to pass a national board exam are not the
book. This is a comprehensive board review designed to help only ones who might benefit from this book. Anyone wanting
the reader study for the general pediatrics board examination. to learn more about pediatric medicine should read this book.
This text covers all the content that the American Board of It provides a broad overview perfect for both early and seasoned
Pediatrics (ABP) says you need to know for the board exam. The learners.
35 chapters in this text correspond to the 3 5 sections of the ABP We are very proud of the final product and believe it pro­
content specifications and are written by specialists in the topic vides the reader with an exceptional resource to cover the entire
areas. The majority of the content is presented as cases followed breadth of pediatric medicine.
by question/answer/discussion. The discussions are in depth but We would like to thank all of the authors who contributed to
written in an informal manner to avoid the feeling that you are this book. Without their hard work, this book would have never
reading a textbook. For visual learners, we have included tables, come together. We would also like to thank Christie Naglieri,
figures, and photos. The goal of this book is to make studying Andrew Moyer, Alyssa Fried, and Samantha Williams at
for the boards more engaging. McGraw-Hill for their help and guidance throughout. But most
Each chapter is meant to stand alone, allowing you to focus of all, we would like thank our friends, family, and coworkers for
on challenging content areas or those where you may need to their love, understanding, and support during the final push to
spend more time. A final exam is included at the end to help complete this project.
you test what you have learned. Each question is referenced in
the book so you can go back and review what you may have Andrew R. Peterson, MD, MSPH
missed. Kelly E. Wood, MD

XIII
Adolescent Medicine and Gynecology 1
Gaya t h r i C h e l va k u m a r a n d Pa u l a Cody

staging, after the pediatrician who first described the sequence


of secondary sexual characteristics. (See Figures 1 -2 and 1 -3 . )
A 1 4-year-old boy presents to your office with concerns Th e delay leads t o a comparative decrease i n growth velocity
of delayed puberty. The patient is shorter than most of compared to age-matched peers, leading to short stature as the
his classmates. He is active in basketball and is worried primary complaint in most patients. Prepubertal growth veloc­
that his lack of height will affect his ability to play. His ity is typically 4 to 6 cm/y in adolescent boys and increases to
mother is 5 feet, 7 inches tall and had her first menses at a peak velocity of approximately 9.5 cm/y at SMR 3 to 4. Most
age 1 4 years. His father is 6 feet, 4 inches tall and reports delayed puberty in boys is due to a constitutional delay from
that he was a "late bloomer" and attained most of his delayed activation of the hypothalamic-pituitary-gonadal axis.
adult height in college. The patient is otherwise healthy, Once puberty begins patients generally have catch-up growth
developmentally appropriate, and not on any medications. and attain a normal adult height. Often there is a family his­
On physical exam he is a well-appearing, well-nourished tory of "late bloomers" or other family members with constitu­
young male. He has mild acne, his testes are descended tional delay of puberty. Delayed puberty and short stature can
bilaterally and 3 mL in volume, and there is scant pubic have significant effects on self-esteem, particularly in boys, and
hair and minimal penile development. He has grown 5 em short courses of androgen replacement therapy may be indi­
in the last year. (See Figure 1 - 1 .) cated. (Think back to junior high when the girls towered over
the boys.) Underlying metabolic, endocrine, or systemic dis­

Question 1 - 1
orders are an unlikely cause of delayed puberty in this patient
given his previous normal growth velocity and development
Which of the following tests will most likely establish the and otherwise healthy state. A bone age test will help establish
diagnosis for this patient? the diagnosis. In constitutional delay of puberty bone age will be
A) CBC. decreased compared with chronological age as it is more closely
B) Calculation of midparental height. related to skeletal maturity and pubertal stage. A normal bone
C) Bone age. age would be seen with familial short stature and Turner syn­
D) Growth hormone levels. drome. An advanced bone age is seen with precocious puberty
E) Thyroid studies. for which early closure of the growth plates will result in a short
adult unless treated.
Discussion 1 - 1
The correct answer is "C' The age at which puberty is considered
delayed is 14 years in boys and 1 3 years in girls. Constitutional • Helpful Tip
delay of puberty is the most common cause, especially in boys. � Ca lculation of a m i d p a renta l height ca n help determ i n e
This patient most likely has a constitutional delay of puberty­
short but normal growth rate and a positive family history. The
1 1 1 r a c h i l d's genetic height potentia l .
F o r g i rls: (Mother's height i n e m + Father's height i n
best test to establish the diagnosis would be a bone age. Delayed cm}/2 - 6.5 em
puberty in boys is defined as lack of pubertal testicular devel­ For boys: (Moth er's height i n em + Father's height i n
opment (sexual maturity rating [SMR] 2) by age 14 years in cm}/2 + 6.5 em
boys. (See Table 1 - 1 for details.) SMR is also known as Tanner
2 MCGRAW-HILL EDUCATION SPECIALTY BOARD REVIEW: PEDIATRICS

2 to 20 years: Boys NAME


Stature-for-age and Weight-for-age percentiles RECORD #
12 13 14 15 16 17 18 19 20
Mother’s Stature — Father’s St::ure — AGE cm
ature
1

1 sT
A
T
Calculate BMI: Weight (kg) + Stature (cm) + Stature (cm) x 10,000 U
or Weight (lb) + Stature (in) + Stature (in) x 703 R
in on: 10 11 E
1

62 1
S
T
A
T
U
R
E

W
E
I
G
H
T

W
E
l
G
H
T

AGE
2 3 4 5 6 7 8 91011121314151617181920
Published May 30, 2000 (modified 11/21/00).. 7,77"
SOURCE: Developed by the National Center for Health Statistics in collaboration with I I
the National Center for Chronic Disease Prevention and Health Promotion (2000). III/ll 4?
http:llwww.cdc.gov/growthcharts SAFER . HEALTHIER- PEOPLE”

FIGURE 1-1. Growth chart of boy in Case 1. (Reproduced with permission from the National Center for Health Statistics in collaboration with the National
Center for Chronic Disease Prevention and Health Promotion [2000]. http://www.cdc.gov/growthcharts.)
CHAPTER 1 • A D O L E S C E N T M E D I C I N E A N D GYN ECOLOGY 3

TA B L E 1-1 S EXUAL MATU RITY RAT I N G ( S M R ) I N MALES

SMRin
Males Pubic Hair Development Testicular Development Penile Development
1 No p u b i c h a i r Prepu berta l genita l i a Prepu berta l genita l i a
2 Spa rse, downy h a i r at base of E n l a rgement o f testis (vo l u m e > 4 ml), No c h a n g e
pe n i s scrota l sac e n l a rg es, red der in a ppea ra nce
3 H a i r beco mes thi cker, longer, Conti n ued e n l a rgement of testes a n d Pen i s beg i n s t o g row i n
a n d c u r l i er, sti l l i n l i m ited scrot u m length fi rst, then d i a m eter
m i d l i ne d i stribution
4 Ad u lt type hair in q u a l ity but Th icke n i n g a n d d a r ke n i n g o f scrota l sac Cont i n ued g rowth of pe n i s,
l i m ited d i stribution with conti n u ed g rowth of testes e n l a rgement of g l a n s
5 Ad u lt q u a l ity hair with spread Ad u lt a p peara n ce, a d u lt testi c u l a r vol u m e Ad u l t a p peara n ce
to med i a l t h i g h s o f 1 2-27 m l

Data from Bord i n i B, Rosenfield R. Normal pu berta l development pa rt I I : C l i n ica l as pects of puberty. Pediatr Rev. 2 0 1 1 ;32(7):2 8 1 -292; a n d
Nei n stein LS, e d . Handbook o fAdolescent Healthcare. P h i l a d e l ph ia, PA: Lippi ncott Wi l l ia m s & Wi l ki n s; 2009.

tender 0.5 em mass under both nipples. His testicular


exam reveals testicular volume of 6 mL and no masses.
Your next patient is a 1 3 -year-old boy who is distressed The remainder of his exam is normal, his growth and
because he reports that he is developing breasts. On development is otherwise normal, and he is not on any
exam you note that he has a firm, rubbery, mobile, and medications.

GIRLS

HEIGHT SPU RT
Height 3 in/y
G ROWTH RATE Weight 17.5 lb/y
AGE RANGE
Height 2 in/y ... 11.5-16.5 y
Weight 6 lb/y AGE RANGE 10-16.5 y
MENARCHE Average height 62.5 in (158.5 em)
Average weight 106 lb (48 kg)

B REAST

ffi ·1� � ��� [�1M bi�----


B reast buds
begin.
B reast and
areola grow.
Nipple and
areola form
Areola rejoins
breast contour
AGE RANGE separate and development
8-13 y mound , pro- is complete.
truding from AGE RANGE
breast. 12.5 -18.5 y

SEXUAL MATU RITY 2 3 4 5


RATING

I i r I�! r I� l r ll I l r 11
- - �

PUBIC HAI R

I n itial hair is Pubic hair Hair looks like I nverted triangular


straight and fine. becomes an adult's but limited pattern is established
AGE RANGE coarse, darkens, in area. AGE RANGE
8-14 y and spreads. 12.5-16.5 y
AGE 11 y 12 y 13 y 14 y 15 y
FIGURE 1-2. Adolescent fem a l e sexual matu ration a n d g rowth. (Reproduced with permission from Hay WW, Levin MJ, Deterd i n g RR, Abzug MJ, eds. Current
Diagnosis and Treatment Pediatrics. 22nd ed. New York, NY: McGraw- H i l l Education, I nc., 20 1 4; Fig. 4-4.)
4 MCGRAW- H I L L E D U CAT I O N S P E C I A LTY BOARD REVI EW: P E D I ATRICS

BOYS APEX ST RE NGTH SPURT


HEIGHT SPURT Height spurt 10-12 in (25-30 em)
Weight 44 lb (20 kg)

Height 4 in/y
G ROWTH RATE AGE RANGE
Weight 20 lb/y
Height 2 in/y 13-17.5 y
Weight 6.5 lb/y

lh\1 llf\�IY\�Ii\�
PE NIS
TESTES

Testes increase in size Penis grows in Penis grows in Development


and skin of scrotum length. width. is complete.
reddens. AGE RANGE
AGE RANGE 14.5-18 y
10-13.5 y
SEXUAL MATU RITY .......,,----.;;.2-,-...,
3 4 5
RATI NG

PUBIC HAI R
Straight hair Hair becomes Hair is full, Full development.
appears at curly, coarse, limited in area.
penis base. and dark. AGE RANGE
AGE RANGE 14.5-18 y
10-15 y
AGE 11 y 12 y 13 y 14 y 15 y 16 y 17 y
FIGURE 1-3. Adolescent m a l e sexual maturation a n d g rowt h . (Reproduced with permission from Hay WW, Levin MJ, Deterd i n g RR, Abzug MJ, eds. Current
Diagnosis and Treatment Pediatrics. 22nd ed. New York, NY: McGraw- H i l l Education, I nc., 20 1 4; Fig. 4-3.)

Question 2-1 Question 2-2


The most likely diagnosis for this patient is: Management for this patient would include:
A) Pseudogynecomastia. A) Testicular ultrasound.
B) Testicular tumor. B) Reassurance and follow-up exam in 6 months.
C) Gynecomastia. C) Measurement of gonadotropins.
D) Normal puberty. D) Measurement of prolactin.
E) Phytoestrogen consumption. E) Testosterone injections.

Discussion 2-1
The correct answer is "C:' Gynecomastia is a common con­ Discussion 2-2
dition in adolescent males with a prevalence of 19.6% in The correct answer is "B:' Given that this patient has a nor­
1 0 . 5 -year-old males, increasing to 64.6% by age 14 years. It is mal testicular exam and no signs of exogenous estrogen
caused by an imbalance of estrogen to testosterone in pubertal exposure or underlying disease, reassurance and follow up is
males. The relative increased estrogen leads to proliferation of appropriate.
glandular breast tissue. It is important to differentiate gyneco­
mastia from pseudogynecomastia, which results from excess
fat deposition as opposed to glandular tissue. In pseudogyne­
comastia, tissue tends to be more widely distributed and not � QUICKQUIZ
localized to the nipple areolar complex. This patient has a nor­
mal testicular exam, making a testicular cancer unlikely. But You are seeing a 1 3-year-old boy who has testicular volume of
take the opportunity to remind him to perform monthly self­ 6 mL and light downy pubic hair.
exams as testicular cancer usually presents as a painless mass. What is his SMR staging?
Soy products contain phytoestrogen, but eating tofu won't give A) Testicular volume SMR 2; pubic hair SMR 2.
you breasts. Have you ever seen an orchidometer? To avoid B) Testicular volume SMR 1 ; pubic hair SMR 2.
confusion, be sure to explain that it is standard practice to C) Testicular volume SMR 2; pubic hair SMR 3 .
compare the patient's testicles to wooden beads on a string to D) Testicular volume SMR 3; pubic hair SMR 3 .
determine the testicular volume. E) Testicular volume SMR 2; pubic hair SMR 1 .
CHAPTER 1 • A D O L E S C E N T M E D I C I N E A N D GYN ECOLOGY 5

Discussion TA B L E 1-2 S EXUAL M ATU RITY RAT I N G ( S M R )


The correct answer is ''A:' I N FEMALES

SMRin Pubic Hair Breast


• Helpful Tip Females Development Development
:5.� The average age of menarche in the U n ited States is
i1 1r 1 2.6 yea rs, with ra nge o f 1 1 .0 t o 1 4. 1 . Menarche occu rs No pu bic h a i r Prepu be rta l brea sts
ea rlier in Africa n American a n d Mexica n American g i rls. 2 Spa rse, d owny Fo rmation of breast
hair b u d , g l a n d u l a r tissue
palpable u n d e r a reola;

� QUICKQUIZ
a reola i s s l i g htly wid-
ened a n d projects as a
small mound
What is the most common breast mass in adolescent females? 3 H a i r beco mes E n l a rgement of breast
A) Fibroadenoma. thicker, longer, with e l evation of
B) Fibrocystic changes. a n d c u r l ier, sti l l breast conto u r a n d
C) Rhabdomyosarcoma. i n l i m ited m i d - e n l a rgement o f a reola
D) Hemangioma. line d i stri bution
E) Galactocele. 4 Ad u lt type Areo la fo rms a sec-

Discussion
h a i r in q ua l - o n d a ry m o u n d ove r
ity but l i m ited conto u r of breast
The correct answer is "/\'. Most breast masses in adolescent d i stribution
girls are benign. A fibroadenoma feels like a rubbery, smooth,
5 Ad u lt q u a l ity Fu l ly mature breast
mobile, round mass. It is nontender and usually located in the
h a i r with spread with conti n u o u s con-
upper outer quadrant of the breast. Other common benign
to med i a l t h i g h s to u r between a reola
masses include fibrocystic changes, cysts, abscesses, and fat
a n d breast
necrosis from trauma. Options "D" and "E" are less common
benign causes. Malignancy such as option "C" is a rare cause. Data from Bord i n i B, Rosenfield R. Normal pu berta l development
Before a nipple is pierced or a hair is plucked, remember to pa rt II: C l i n ical a spects of p u berty. Pediatr Rev. 2 0 1 1 ;32(7):28 1 -292;
counsel that both can cause an abscess. and Neinstein LS, ed. Handbook ofAdolescent Hea/thcare. Philadelphia,
PA: Lippincott Wi l l ia m s & Wil kins; 2009.

Question 3-2
You are seeing a 14-year-old girl in your office for her
annual exam. When speaking with you confidentially, she
mentions that she is concerned that she has not yet started The first sign of puberty in females is typically:
her period like all of her friends. She reports breast devel­ A) Breast development.
opment starting approximately 1 year ago. On exam she has B) Development of pubic hair.
palpable breast tissue extending just beyond her areola and C) Menses.
pubic hair that is thick and curly and primarily midline in D) Body odor.
distribution. E) Acne.

Discussion 3-2
Question 3-1
The correct answer is ''A:' Breast development (thelarche) is
Which of the following most accurately describes her SMR
typically the first sign of puberty in girls and typically occurs
staging?
between age 8 and 1 3 years (see Figure 1 -2 ) . Breast develop­
A) Breast SMR 1 ; pubic hair SMR 2.
ment typically precedes pubarche (pubic hair development)
B) Breast SMR 3 ; pubic hair SMR 2.
though in some girls pubarche may occur first or simulta­
C) Breast SMR 1 ; pubic hair SMR 4.
neously. Pubarche typically occurs 1 to 1 .5 years after breast
D) Breast SMR 3 ; pubic hair SMR 3.
development. Menarche occurs approximately 2.5 years after
E) Breast SMR 5; pubic hair SMR 5.
thelarche at an average age of 1 2 . 6 years in Caucasians and
earlier in African Americans and Mexican Americans. Girls
Discussion 3-1 reach their peak height velocity of 8.25 cm/y earlier than boys
The correct answer is "D:' The patient described in the vignette at approximately SMR 3 . Peak height velocity in girls always
has breast and pubic hair development consistent with SMR 3 . precedes menarche. Peak height velocity occurs at approxi­
(See Table 1 -2 for details.) mately SMR 4 to 5 in boys.
6 MCGRAW- H I L L E D U CAT I O N S P E C I A LTY BOARD REVI EW: P E D I ATRICS

• Helpful Tip • Helpful Tip


� The fi rst stage of puberty for m a l es is testi c u l a r � Don't forget that isolated G n RH d eficiency has been
1 1 1r enla rgement, defi ned as a testis vol u m e of 4 m L 1 1 1r associated with both Ka l l ma n syn drome a n d anosmia.
or g reater, or 2 . 5 e m i n d i a m eter. F o r g i rl s, i t is the
a ppea ra nce of b reast buds.

� QUICKQUIZ A mother brings her 1 7-year-old son for his annual health
maintenance exam. When you ask if she has any concerns
Which of the following is correct? about him, she mentions that he sleeps all the time. She
A) Adrenarche results from testosterone secretion by the also states that he was always a happy child but recently has
gonads. become more withdrawn and always seems tired. She reports
B) Activation of the hypothalamic-pituitary-gonadal axis that his grades have been declining, and he does not seem to
(HPA) causes gonadarche and adrenarche. enjoy activities he previously enjoyed, such as playing soccer
C) Estrogen secretion causes armpit hair development. and video games with friends.
D) All of the above.
E) None of the above. Question 4- 1
The next step in diagnosing this patient is:
Discussion A) Obtaining a CBC and iron studies.
The correct answer is "E:' Puberty encompasses gonadarche and B) Obtaining a complete psychosocial history from the patient.
adrenarche. Both are separate events, but the timing typically C) Thyroid testing.
overlaps. Gonadarche, growth and maturation of the gonads D) Intelligence testing.
(testes, ovaries) , is under the control of the HPA secretion of E) Completion of Vanderbilt forms by parents and teachers.

Discussion 4- 1
gonadotropin-releasing hormone (GnRH). Before puberty,
release of GnRH from the hypothalamus is inhibited. Adrenal
androgen secretion (dehydroepiandrosterone [DHEA] and The correct answer is "B:' Adolescence is time of rapid growth
androstenedione) causes pubic and axillary hair development, and development. It can be a stressful time, manifesting as anxi­
acne, and body odor (adrenarche) . ety, withdrawal, aggression, somatic complaints, depression, or
poor coping skills such as using drugs. The most common causes
of morbidity and mortality in adolescence are related to the
risk-taking behavior and experimentation that is a normal part
• Helpful Tip
� Detection of noctu rnal l utei n izing hormone (LH) p u l ses
of adolescent development. Obtaining a thorough psychosocial
history is important in screening for these risk-taking behaviors
I llr is the fi rst hormonal s i g n that puberty has sta rted . At and identifying protective factors. The symptoms the mother
pu berty, the hypoth a l a m u s is no longer i n h i bited a n d
has described raise concern about depression in this patient. A
releases G n R H i n a p u l satile fas h ion. G n R H sti m u lates
thorough psychosocial assessment using the HEADSSS screen­
the a nterior pitu ita ry to secrete gonadotropins-LH
ing tool with a follow-up depression screen will likely reveal the
fi rst, then fol l icle-sti m u lating hormone (FSH). FSH and
cause of his symptoms. How many knew that Vanderbilt scales
LH sti m u l ate the gonads to produce ga metes (eg g s or
assess for attention deficit hyperactivity disorder (ADHD) ?
sperm) a n d sex hormones (estra d i o l or testosterone).

Question 3-3 • Helpful Tip

What is the next step in management of this patient? � H EADSSS was developed as a psychosocial scree n i n g

A) Bone age. 1 llr tooL

B) Measurement of gonadotropins. H- Home (Who l ives with the teen? How does the teen
C) Reassurance and follow up in 6 months to a year if no menses. get a l o n g with fa m i ly?)
D) Thyroid studies. E - Education (Is the teen in schoo l ? How is he or s h e
E) CBC. perform i n g i n s c h o o l ? S c h o o l performa nce can be a n
i m porta nt i n d ication o f h o w a teen is fu nctio n i n g .)

Discussion 3-3 E- Eati ng (meal consi stency; body image)


The correct answer is "C:' This patient is progressing through A - Activities
the stages of puberty and will likely attain menarche in the next D- Drugs (a lcohol, tobacco, m a rij u a na, and other drug
6 months to 1 year. If no menses occur by age 1 6 further workup use, i n c l u d i n g prescription a n d over-the-cou nter)
would be warranted.
CHAPTER 1 • A D O L E S C E N T M E D I C I N E A N D GYN ECOLOGY 7

Question 4-3
S Sexua l ity (Sexual attraction: Are you attracted
-
Which of the following is true about sexually transmitted
to m a l es, fem a l es, both, neither? Sexua l behavior: infection (STI) transmission?
Have you ever had sex, how m a ny partners, h i story of A) HIV transmission rates are low with receptive anal
sexu a l l y tra n s m itted i nfection d i a g n osis and testi ng, intercourse.
condom use, contraceptive u se, last sexua l activity, B) STis cannot be transmitted through oral sex.
h i story of forced sex?) C) Women who have sex with women are at a low risk for STis.
S- Su icide/Depression D) Condoms are effective at reducing STI transmission.
S - Safety (Does the teen feel safe at home or school?
What is h i s or her exposure to violence?) Discussion 4-3
The correct answer is "D:' HIV transmission rates are high
with receptive anal intercourse due to microtrauma during
intercourse. STis can be transmitted through oral sex, and it is
Through the HEADDSS assessment you learn that the patient important to educate patients to use condoms when having oral
is attracted to males and recently entered a relationship with sex to reduce the risk of contracting an STI. Studies have shown
a boy at school. He wrote a letter to the boy which another that women who have sex with women are at an increased risk
student found and shared with the whole class. Since then the of contracting human papillomavirus (HPV) , trichomoniasis,
patient reports that he is teased by many of his classmates and and HIV Condoms are an effective method of STI prevention
has been skipping classes to avoid being teased. His depres­ and when properly used have been shown to reduce rates of
sion screen is positive for sadness, anhedonia (no pleasure in transmission of HIV, gonorrhea, chlamydia, trichomonas, and
activities), excessive sleeping, and feelings of guilt. He denies hepatitis B. They can also be effective at preventing STis trans­
any thoughts of self-harm or suicidality. mitted by skin-to-skin or mucosal contact, such as herpes sim­

Question 4-2
plex virus (HSV) , syphilis, and HPV, but only if the affected area
is covered by the condom. Equally important is making sure
Adolescents who identify as lesbian, gay, or bisexual are at adolescents know how to put on a condom. Pregame practice
increased risk for which of the following? is a good idea.
A) Eating disorders.
B) Substance abuse. Question 4-4
C) Depression. You also counsel the patient that the most common cause of
D) Bullying. mortality in the adolescent population is:
E) All of the above. A) Cardiac disease.

Discussion 4-2
B) Unintentional injuries.
C) Suicide.
The correct answer is "E:' Sexual development is one part of ado­ D) Homicide.
lescent development. During early adolescence pubertal develop­ E) Cancer.
ment is just beginning. At this stage of development adolescents are
very focused on changes occurring in their bodies and question­ Discussion 4-4
ing whether they are normal. Adolescents may begin to experience The correct answer is "B:' The leading cause of death in the ado­
sexual fantasies and experience sexual pleasure through masturba­ lescent population is unintentional injuries, with motor vehicle
tion. Sexual intercourse at this stage is uncommon, but may occur. collisions being the most frequent cause of such injury in this
Adolescents often experience crushes, which may be same sex or population. (See Figure 1 -4.) Homicide is the second leading
opposite sex. These patterns of attraction may or may not persist cause of death, followed closely by suicide. Organic disease is
into future stages. In middle adolescence physical development is a less frequent cause of mortality in this age group. Screening
nearing completion; at this stage adolescents are forming their sex­ for risk factors such as substance use, mental illness, and expo­
ual orientation and identity. Sexual experimentation is common at sure to violence is important in this population to address the
this stage and many adolescents may have intercourse for the first leading causes of mortality. Good driving habits, including no
time. By late adolescence the goal is to become a sexually healthy texting while driving, should be discussed.
adult with the ability to form long-lasting relationships. Sexual
orientation refers to an individual's pattern of physical and emo­
tional attractions to others and involves complex components such
as fantasies and feelings. Personal, family, cultural, developmen­ � QUICKQUIZ
tal, and social factors can affect an individual's ability to identify,
accept, and act on his or her attractions. Adolescents who identify Which of the following is a risk factor for suicide?
as lesbian, gay, or bisexual are at an increased risk for a number of A) Bullying.
conditions, including eating disorders, substance abuse, and men­ B) Witnessing violence.
tal health illnesses, particularly depression and anxiety. C) Social isolation.
The patient has questions about how to stay safe when he D) Mental illness.
does become sexually active with his partner. E) All of the above.
8 MCGRAW- H I L L E D U CAT I O N S P E C I A LTY BOARD REVI EW: P E D I ATRICS

U n i ntentional i n j u ry 1 -
- ---------------'
]

A 1 6-year-old girl and her mother present to your office with


Homicide I concerns about irregular periods. The patient had her first

I
menses at 12 years of age and had regular monthly periods
Suicide
until 6 months ago when her periods stopped. She has had an

Malignant neoplasms t:::J accompanying 50-pound weight loss over the past 6 months.
When asked further about the weight loss, she reports that

H eart d isease � she has been working on more healthful eating, has cut all
desserts and junk foods out of her diet, and eats a low-fat and
0 2000 4000 6000 8000 1 0000 1 2000 1 4000 low-carb diet. In addition she has started running 3 miles a
day in order to "get healthy?' On physical exam her vital signs
FIGURE 1-4. 20 1 0 Lea d i n g causes of death in youth ages 1 5 to 24 years of
age in the U n ited States. (Reproduced with permission from the Centers for are temperature 36.4°C {97.5°F) , heart rate 44 beats per min­
Disease Control a n d Prevention, National Center for I nj u ry Prevention a n d ute, blood pressure 96/60 mm Hg, and respirations 16 breaths
Control, Web-based I nj u ry Statistics Query a n d Reporting System (WI SQARS). per minute. She appears thin, with sallow-looking skin and
Accessed J a n u a ry 28, 20 1 5 from http://www.cdc.gov/inj u ry/wisqa rs/.) dry hair. She is bradycardic on exam, with no murmurs and
a regular rhythm. Her heart rate increases by 19 beats during

Discussion
positional changes from sitting to standing, with minimal
change in her blood pressure. Her pulses are strong and sym­
The correct answer is "E:' Additional risk factors include family metric while her fingers and toes are cool to touch.
history of suicide, history of abuse, previous attempt, access to
means such as firearms, alcohol and drug use, stressful events, Question 5-1
and sexual identification other than heterosexuality. Which of the following is the most likely cause of this patient's
symptoms?
The patient's mother mentions that she recently caught him A) Thyroid disease.
smoking pot in the garage with some friends. She is request­ B) Anorexia nervosa.
ing that you drug test him without letting him know. C) Bulimia nervosa.
D) Diabetes mellitus.
Question 4-5 E) Coarctation of the aorta.
Your next step is to:
A) Do as the mother requests. Discussion 5-1
B) Notify the patient and perform testing regardless of his The correct answer is "B:' Eating disorders are a common but
wishes. often underdiagnosed condition in the pediatric population. The
C) Notify the patient and perform the testing if he agrees. 12-month prevalence of anorexia nervosa among young females
D) Refuse to perform drug testing. in approximately 0.4%, the prevalence of bulimia nervosa is
E) Reassure the mother that catching him guarantees he will approximately 1% to 1 .5%. Anorexia nervosa has a mortality rate
stop using. of 5% to 6%, the highest of any psychiatric illness. Patients with
anorexia nervosa generally present with rapid weight loss sec­
Discussion 4-5 ondary to caloric restriction, which may present as elimination
The correct answer is "C:' Recreational drug use is an under­ of "junk food" from the diet; avoidance of certain food groups,
recognized cause of morbidity and mortality in adolescents. such as carbohydrates and fats; or changing to a restrictive vegan
Indications for drug testing in the acute care setting include or vegetarian diet. Patients may also try and reduce weight by
acute presentation with altered mental status, suicide attempt, over-exercising or purging through self-induced vomiting or
unexplained seizures, syncope, arrhythmias, or the pres­ use of diuretics and laxatives. In contrast, patients with bulimia
ence of toxidromal signs. In the primary care setting volun­ nervosa typically present with cycles ofbinging that trigger purg­
tary drug testing can be helpful for assessment, therapy, and ing or inappropriate compensatory behaviors. Compensatory
monitoring. The American Academy of Pediatrics (AAP) cur­ behaviors could include self-induced vomiting, use of diuretics,
rently cautions against involuntary drug testing of adolescents use of laxatives, fasting, or over-exercising. Patients with bulimia
in nonemergent settings. Testing of competent adolescents nervosa are typically of normal weight or overweight. Patients
without their knowledge is unethical and illegal, and without with anorexia nervosa often present with signs of malnutrition
their consent is impractical. If a pediatrician suspects that a (eg, bradycardia); hair, skin, and nail changes, often manifesting
patient is abusing drugs and the patient refuses drug testing, as dry and brittle hair, nails, and skin; menstrual irregularities ( eg,
documentation of the refusal and referral to a mental health or amenorrhea and oligomenorrhea); orthostatic vital sign changes;
addiction specialist may be warranted. Given the limitations cold intolerance; acrocyanosis; mood changes; and fatigue. The
of currently available drug tests, a thorough substance abuse DSM is revised periodically and eating disorder diagnostic cri­
history often provides more useful information on drug abuse/ teria often change. Due to copyright restrictions, we are unable
use than a drug test. to print the DSM-5 diagnostic criteria for anorexia and bulimia.
CHAPTER 1 • A D O L E S C E N T M E D I C I N E A N D GYN ECOLOGY 9

TA B L E 1-3 COMMON F EATURES OF ANOREXIA


For bulimia nervosa:
N E RVOSA • Syncope
• Restricted eati ng • Hypokalemia
Low body wei g h t for age a n d sex • Severe hypochloremia ( <88 mmol!L)

• Fea r of g a i n i ng wei g h t or beco m i n g fat


• Esophageal tears
• Arrhythmia
• Behaviors that i nterfere with g a i n i ng wei g ht
• Hypothermia
• Distu rba nce of body i m a g e
• Suicidality
• Excessive i nfl ue nce o f body weight o r s h a pe o n
• Intractable vomiting
self-eva l u ation
• Hematemesis
• Lack of recog n ition of the seriousness of low body weight
• Failure of outpatient treatment

This patient warrants inpatient treatment due to her brady­


Summaries of their common features are included in Tables 1-3 cardia. Note that the AAP endorses electrolyte abnormalities
and 1 -4. But the reader should make themselves familiar with the as admission criteria for bulimia, but not anorexia. In reality,
current DSM criteria for both conditions. a patient with anorexia and severe electrolyte abnormalities
would also likely be admitted to the hospital. Also of interest
Question 5-2 is that hypothermia is defined for one but not both disor­
Which of the following would be a reason for inpatient hospi­ ders . The goal in hospital admission is medical stabilization,
talization for this patient? which is usually achieved through nutritional rehabilitation
A) Hypertension. and correction of any underlying electrolyte abnormalities.
B) Bradycardia. Fluid and electrolyte shifts can occur with the reintroduction
C) Hypokalemia. of nutrition in a patient who has been malnourished. This
D) Rapid weight loss. is called refeeding syndrome and most typically manifests as
E) Amenorrhea. decrease in phosphate, magnesium, and potassium, as well as
an increase in extracellular volume causing peripheral edema
Discussion 5-2 or congestive heart failure, or both. These fluid and electro­
The correct answer is "B:' The AAP suggests that the following lyte shifts can lead to cardiac arrhythmias; thus monitoring
signs and symptoms warrant inpatient hospitalization: of cardiac status, electrolytes, and fluid status is important in
For anorexia nervosa: the inpatient setting.
Bradycardia ( <45 bpm daytime or <45 bpm nighttime)
Question 5-3

• Hypotension ( <90 mmHg systolic)


Which of the following will likely NOT be a part of this
• Arrhythmia
patient's treatment?
• Hypothermia A) Nutritional rehabilitation.
• Weight <75% ideal body weight B) Psychotherapy.
• Body Fat < 10% C) Medical monitoring for complications of illness.
• Refusal to eat D) Use of an appetite stimulant.
• Failure of outpatient treatment E) Family therapy.

Discussion 5-3
TABLE 1-4 COM M O N FEAT U R E S OF B U L I M I A The correct answer is "D:' Treatment of eating disorders is mul­
N E RVOSA ( M U ST B E P R E S E N T AT LEAST O N C E A tidisciplinary and includes medical monitoring for complica­
W E E K FOR 3 MONTHS) tions of the illness such as refeeding syndrome; psychotherapy
to address eating disorder thoughts and body image concerns;
• Eati n g a l a rg e a m o u nt of food ove r a short a m o u nt of and nutritional rehabilitation, which involves reintroducing
time meals and snacks in a stepwise fashion with a goal of restor­
• Lack o f control d u ring t h e episode-feels as though o n e ing body weight. The use of appetite stimulants is not recom­
ca nnot stop eati ng mended as this does not address the disordered thoughts and
• Behaviors to co m pe n sate fo r epi sodes of bi nge eati n g . behaviors that are a part of the eating disorder. Treatment goals
include medical stabilization; nutritional rehabilitation as mea­
• Excessive i nfl u e nce o f body weight o r s h a pe o n
sured by restoration of body weight, usually at a rate of 0.25 to
self-eva l u ation
1 kg per week in the outpatient setting; decrease in eating dis­
• Diagnosis i s not bette r exp l a i ned by a d i agnosis of order thoughts and behaviors; and improvement in body image.
a n o rexia ne rvosa Healthy family involvement is always good.
10 MCG RAW-H I LL E D U CAT I O N S P E C I A LTY BOARD REVI EW: P E D I ATRICS

sit out practice and competitions for the past few weeks, she
• Helpful Tip
� Anorexic patients a re at risk for refeeding syn d rome.
has kept active by riding a stationary bike and swimming.
She is otherwise healthy and takes no medication. The patient's
I llr Hypophosphatemia is the h a l l ma r k a n d the pri m a ry breast development began at about 1 2 years of age and pubic
c u l p rit of refeed i n g syn d rome. hair 2 years ago. Her brother has insulin-dependent diabetes
mellitus and her mother has hypertension. The mother states
that her own menses started at 14 years of age. The patient is

Question 5-4
a sophomore in high school and is getting straight Xs. She has
a boyfriend but denies sexual activity. She denies all substance
Which of the following laboratory findings would NOT be
use. Review of systems is negative for headaches, nausea, vom­
expected in a patient with anorexia nervosa?
iting, abdominal pain, constipation, diarrhea, or vaginal dis­
A) Hyperkalemia.
charge. On physical exam she is 5 feet, 6 inches tall and weighs
B) Leukopenia.
1 10 pounds, with a heart rate of 56 bpm and a blood pressure
C) Normal laboratory results.
of 1 10/70 mm Hg. She is SMR 3 for breasts and pubic hair. The
D) Hypoglycemia.
rest of her exam is normal.
E) Elevated liver enzymes.
Question 6- 1
Discussion 5-4
The definition of primary amenorrhea includes absence
The correct answer is "A:' Most patients with eating disorders
of menses by what age, assuming normal secondary sexual
have normal laboratory findings. General laboratory workup
development?
includes a CBC with white blood cell count differential; full
A) 12 years.
chemistry panel, including liver and renal studies; thyroid stud­
B) 14 years.
ies; nutritional markers, such as vitamin D and prealbumin; and
C) 16 years.
coagulation studies. Don't forget an ECG. Technically it isn't
D) 18 years.
a lab test but we needed to include it somewhere! Additional

Discussion 6-1
testing may be indicated to exclude alternative diagnoses. Com­
mon laboratory abnormalities that may be seen in patients with
anorexia include hypokalemia (not hyperkalemia) and hypo­ The correct answer is "C." Primary amenorrhea refers to the
chloremic metabolic alkalosis secondary to purging. Hypogly­ absence of menses ( 1 ) by age 16 years with normal second­
cemia can be seen secondary to malnutrition. Hyponatremia ary sexual development; (2) by age 14 years in the absence
may be seen as a sign of water-loading or excessive water intake. of any breast maturation; or ( 3 ) despite having attained
Leukopenia can be seen, and in severe cases pancytopenia. SMR 5 for 1 year or more, or despite the onset of thelar­
Mild elevation of liver enzymes can also be seen secondary to che 4 years previously. Focus on remembering the first and
malnutrition. second criteria.

Question 6-2
� QUICKQUIZ The most likely cause of primary amenorrhea in this patient is:
A) Hypothalamic amenorrhea.
What is NOT a complication of anorexia nervosa? B) Hypothyroidism.
A) Diarrhea. C) Hyperprolactinemia.
B) Gastroparesis. D) Hypopituitarism.
C) Osteopenia. E) Imperforate hymen.
D) Brain atrophy.
E) Pericardia! effusion. Discussion 6-2
The correct answer is "A." Causes of primary amenorrhea
Discussion include conditions resulting from central dysfunction (hypo­
The correct answer is "A:' Anorexia causes constipation, not thalamic or pituitary) , ovarian dysfunction, or anatomic
diarrhea (unless the patient is taking laxatives) . abnormalities of the genital tract. Given this patient's clinical
presentation, the most likely diagnosis would be functional
hypothalamic amenorrhea, due to partial or complete inhibi­
tion of GnRH release. This inhibition can be due to nutri­
tional deficiencies, cystic fibrosis, eating disorders, excessive
A 16-year-old Caucasian girl comes to the adolescent clinic for exercise, stress, or severe or prolonged illness. Excessive
the first time because she has never had a menstrual period. exercise is to blame in this teenage girl. She has developed a
She is a runner on her school's cross-country team and recently stress fracture from excessive running. Signs and symptoms
was diagnosed with a stress fracture. Although she has had to of hypothyroidism include constipation, dry skin, weight
CHAPTER 1 • A D O L E S C E N T M E D I C I N E A N D GYN ECOLOGY 11

gain, and increased sensitivity to cold. When hypothyroid­ C) Tanner 3 pubic hair.
ism is present in children and teens, one may also see delayed D) Normal linear growth.
puberty and poor growth, resulting in short stature. Hyperp ­
rolactinemia is usually due to a prolactin- secreting adenoma Discussion
that also may cause headaches, visual changes (bitemporal The correct answer is "C' Patients with complete androgen
hemianopsia) , and galactorrhea. Hyperprolactinemia can insensitivity syndrome are ( 46,XY) and have a defect in andro­
also be caused by physiologic hypersecretion of prolactin gen receptors. Thus they do not develop testosterone-dependent
(pregnancy) , hypothalamic-pituitary stalk damage, certain male sexual characteristics, such as pubic and axillary hair.
systemic disorders, or drug-induced hypersecretion (eg, ris­
peridone) . With hypopituitarism, the pituitary gland fails Question 6-4
to produce or does not produce enough of one or more of What tests should you order to assess primary amenorrhea in
its hormones, and multiple body functions can be affected. this patient in order to confirm your diagnosis?
If the complaint relates to menstruation, always make sure A) Pregnancy test.
everything is anatomically correct. Young women need "girl B) Follicle-stimulating hormone (FSH) and luteinizing hor-
parts" to have periods and an open outflow path (eg, no mone (LH) .
imperforate hymen) . C) Prolactin.
D) Thyroid-stimulating hormone (TSH ) .
Question 6-3 E) All of the above.
If this patient lacked breast development, which of the
following would be included in the diagnosis? Discussion 6-4
A) Agenesis of miillerian structures. The correct answer is "E:' Pregnancy should always be excluded,
B) Complete androgen insensitivity syndrome. even in patients who deny sexual activity. It is useful to obtain
C) Asherman syndrome. FSH and LH levels to differentiate between hypothalamic (low
D) Pure gonadal dysgenesis (46,XX with streak gonads) . or normal LH and FSH) and ovarian insufficiency (elevated
LH and FSH) . A prolactin level would confirm the presence
Discussion 6-3 of a pituitary microadenoma, which might (or might not) also
The correct answer is "D:' With pure gonadal dysgenesis, the cause headache, galactorrhea, and bitemporal hemianopsia
streak gonads are unable to produce sex hormones. In a female (decreased vision in the outer half) . TSH level is used to diag­
with pure gonadal dysgenesis, the ovaries do not produce nose thyroid abnormalities, which may impact menses. Even
estrogen, resulting in absence of breast development. Agenesis though you suspect hypothalamic amenorrhea (suppression
of miillerian structures impacts the formation of the internal from excessive exercise) , you should rule out both hyperprolac­
reproductive tract (uterus, fallopian tubes, and upper third of tinemia and thyroid abnormalities.
the vagina), not the ovaries, and has no effect on breast devel­
opment. Individuals with complete androgen insensitivity syn­ Question 6-5
drome are resistant to testosterone due to defective androgen Which of the following would make Turner syndrome an
receptors. They do not develop testosterone-dependent male unlikely diagnosis in a patient with amenorrhea?
sexual characteristics, and the testosterone produced by the tes­ A) Short stature.
tes is aromatized into estrogen, leading to phenotypically female B) Hypogonadotropic hypogonadism.
appearance with normal breast development. Their genotype is C) Hypergonadotropic hypogonadism.
male ( 46,XY) but their external genitalia look female. Internal D) Webbed neck.
female structures such as a uterus are not present as the tes­ E) Widely spaced nipples.
tes make miillerian-inhibiting substance. Asherman syndrome
occurs when uterine synechiae or adhesions obstruct or obliter­ Discussion 6-5
ate the uterine cavity, leading to amenorrhea. Adhesions may The correct answer is "B:' Tuner syndrome ( 45,XO) is the most
develop after uterine infection or other disruption. Breast devel­ common cause of primary gonadal failure (primary hypogo­
opment is not affected. nadism) in adolescent girls and is characterized by ovarian
dysgenesis (accelerated stromal fibrosis and decreased or
absent oocyte production) , short stature, and a wide variety

� QUICKQUIZ of phenotypical abnormalities (widely spaced nipples, webbed


neck, coarctation of the aorta, lymphedema) , and elevated
gonadotropins. The hypothalamus and pituitary function nor­
Which of the following clinical characteristics would NOT mally but the ovary does not, so FSH and LH are elevated.
be seen in a patient with complete androgen insensitivity Most women with Turner syndrome do not develop breasts
syndrome? or have periods (primary amenorrhea) but some develop
A) Absent menses. normally with secondary amenorrhea as their presenting
B) Normal breast development. symptom. Short stature is a big clue.
12 MCG RAW-H I LL E D U CAT I O N S P E C I A LTY BOARD REVI EW: P E D I ATRICS

C) Menstrual cycles lasting 7 days.


• Helpful Tip D) Menstrual cycles resulting in blood loss of 60 mL.
= � To d ifferentiate between primary a n d secondary
r1 1 r hypogonadism, use FSH a n d LH to guide you r thi n king.
I n hypogonadotropic hypogonadism, the problem is Discussion 7-1
centra l (hypotha l a m us or pitu ita ry, or both) a n d FSH and The correct answer is ''A:' Dysfunctional uterine bleeding (DUB),
LH levels wi l l be low or norma l. I n hypergonadotropic also known as anovulatory abnormal uterine bleeding, is defined
hypogonadism, the problem is the gonads (testes or as menstrual cycles occurring less than 20 days or more than
ova ries) a n d FSH a n d LH wil l be elevated. 45 days apart, and lasting longer than 8 days, or menstrual cycles
resulting in blood loss of greater than 80 mL. DUB is character­
ized as ( 1 ) oligomenorrhea-too few periods (> 45 days apart),
(2) polymenorrhea-too many periods ( < 20 days apart), (3)
You decide to proceed with a progesterone challenge in your
metrorrhagia-too-frequent bleeding irregularly or between
patient, which does NOT result in a withdrawal bleed.
periods, (4) menorrhagia-too-heavy blood loss (> 80 mL or
lasting > 7 days), and (5) menometrorrhagia-too-frequent and
Question 6-6 heavy blood loss. Anovulation is a common cause of DUB in ado­
Which of the following are possible causes for failure of a lescents. Without ovulation, progesterone secretion is disrupted,
progesterone challenge? resulting in estrogen-induced overgrowth of the endometrial lin­
A) Elevated estrogen level. ing. The thickened lining outgrows it blood supply and then is
B) Ovarian cyst. shed irregularly.
C) Transverse vaginal septum.
D) Septate hymen.
E) Bicornate uterus. • Helpful Tip
=t'll Anovulatory cycles a re com m o n in early menstru a l
Discussion 6-6 r1 1r cycles a n d a re cha racterized by l a rge va riations i n
The correct answer is "C:' A progesterone challenge helps deter­ estrogen level s a n d l a c k o f progesterone. During
mines the level of endogenous estrogen and confirm the patency of the first 2 yea rs fol l owing menarche, a n ovulation is
the outflow tract. If no bleeding occurs after a progesterone chal­ associated with 50% to 80% of bleed ing episodes. Two
lenge, either the anatomy is disrupted (imperforate hymen, vaginal to 4 yea rs after menarche, a novu lation is associated with
atresia, vaginal septum, miillerian agenesis, absent uterus) or there 30% to 55% of bleed ing episodes. Fou r to 5 yea rs after
is not enough circulating estrogen. Estrogen stimulates buildup of menarche, 20% of bleed ing episodes a re a n ovu latory.
the uterine lining, which is shed in the progesterone challenge. A
transverse vaginal septum is a thin horizontal membrane in the
vagina that may cause obstruction. A septate hymen has an extra
band of tissue in the middle that causes two vaginal openings but • Helpful Tip
not obstruction. A bicornate uterus is just shaped funny. =t'll The fem a l e ath l ete triad i n c l udes osteoporosis,
r1 1r a m e norrhea, a n d d isorders of n utritional i nta ke.

A urine pregnancy test was negative. Based on the clinical


A 1 7-year-old girl presents to the adolescent clinic complain­ picture, you suspect polycystic ovary syndrome (PCOS) .
ing of irregular periods for the past 6 months. Menarche was at
1 1 years of age. Menses have been regular, coming every month
Question 7-2
and lasting 5 days for the past 4 years until 6 months ago. In
the past 6 months, she has had only 2 periods, the last one was
What test should you order to help confirm the diagnosis?
2 weeks ago and lasted 10 days. She has gained 25 pounds in the
A) CBC.
past year (current BMI is 33 kg/m2 ) . She has noticed increasing
B) Estradiol level.
acne and dark, velvety skin in her neck and axilla. She is sexu­
C) Total and free testosterone.
ally active with boyfriend of 1 year and uses condoms consis­
D) FSH level.
tently. The last sexual activity was about 3 weeks ago.
E) Ovarian ultrasound.

Question 7-1 Discussion 7-2


Which of the following qualifies as dysfunctional uterine The correct answer is "C:' Obesity, acanthosis nigricans (see
bleeding? Figure 1 -5), and menstrual irregularities suggest PCOS. There are
A) Menstrual cycles occurring 19 days apart. several diagnostic criteria for PCOS. Essentially, there needs to
B) Menstrual cycles occurring 35 days apart. be evidence of ovulatory dysfunction and clinical or biochemical
CHAPTER 1 • A D O L E S C E N T M E D I C I N E A N D GYN ECOLOGY 13

increased malignancy risk is the prolonged exposure of the


endometrium to unopposed estrogen that results from anovula­
tion. Women with PCOS are not at increased risk of ovarian,
breast, or cervical cancer.

Question 7-5
Other conditions that may show evidence of excess androgen
include:
A) Adrenal tumor.
B) Late-onset congenital adrenal hyperplasia.
C) Anabolic steroid use.
D) All of the above.

Discussion 7-5
The correct answer is "D:' Adrenal tumors can manifest as
FIGURE 1-5. Aca nthosis n i g ricans on the back of the neck. (Reprod uced virilization and can be diagnosed by means of an elevated
with permission from Hoffm a n BL, Schorge J O, Schaffer Jl, H a lvorson LM, dehydroepiandrosterone sulfate (DHEA-S) level. Late-onset
Bradshaw KD, C u n n i n g h a m FG, Ca lver LE, eds. William's Gynecology. 2nd ed. congenital adrenal hyperplasia may become apparent during
New York, NY: McGraw-H i l l Education, I nc., 2 0 1 2; Fig. 1 7-6.) mid-childhood and can lead to early pubic hair, accelerated
bone age, hirsutism, and possible mild clitoral enlargement.
This can be diagnosed based on elevated serum 1 7 -hydroxypro­
signs of androgen excess. Ultrasound evidence of polycystic ova­
gesterone. Anabolic steroids have effects similar to testosterone
ries is suggestive but not required for the diagnosis. Women with
in the body.
PCOS often have an increased level of both total testosterone and
free testosterone. FSH level will not be helpful on its own; how­
ever, in combination with LH level it may be helpful as half of
patients with PCOS have an LH:FSH ratio of 2.5: 1 . Most women � QUICKQUIZ
with PCOS have normal estradiol levels.
Which is a clinical feature of PCOS?
Question 7-3 A) Acanthosis nigricans.
Additional laboratory studies that should be ordered in B) Hirsutism.
patients with PCOS include: C) Acne.
A) Glucose. D) Obesity.
B) Lipid panel. E) All of the above.
C) Insulin.
Discussion
D) Glucose tolerance test.
E) All of the above.
The correct answer is "E:' Obesity, menstrual irregularities, and

Discussion 7-3
infertility are common. Hirsutism, male pattern alopecia, and
acne result from androgen excess. Acanthosis nigricans, as seen
The correct answer is "D:' PCOS patients are at higher risk for
in the patient in this case, is a manifestation of insulin resistance.
impaired glucose tolerance and insulin resistance, as well as
metabolic syndrome, which is a group of cardiovascular risk
factors that include dyslipidemia, type 2 diabetes mellitus,
hypertension, and obesity.

Question 7-4 The mother of a 14-year-old girl calls the clinic stating that
The type of cancer that people with PCOS are at risk for her daughter's first period began 2 weeks ago and she con­
includes: tinues to bleed heavily. Her daughter is complaining of feel­
A) Endometrial cancer. ing increasingly tired. Review of the girl's chart shows that
B) Ovarian cancer. the 14-year-old was previously healthy. She has never had
C) Breast cancer. any surgical procedures. The family history indicates that
D) Cervical cancer. both the mother and maternal grandmother had hysterecto­
E) None of the above. mies in their 30s due to heavy menstrual bleeding. You tell
the mother to bring her daughter to the clinic. At arrival, the
Discussion 7-4 girl's vital signs are heart rate of 1 20 bpm and blood pressure
The correct answer is "A:' Women with PCOS have an increased of 1 00/70 mm Hg. On exam, her skin is pale and vaginal exam
risk of developing endometrial cancer. A major factor for this shows active bleeding from the cervix.
14 MCG RAW-H I LL E D U CAT I O N S P E C I A LTY BOARD REVI EW: P E D I ATRICS

Question 8- 1 C) Start a combined oral contraceptive pill daily.


Which of the following conditions would be an unlikely cause D) Admit to hospital for higher doses of estrogen and possible
of abnormal vaginal bleeding in this patient? blood transfusion.
A) Endometrial cancer. E) Uterine dilational and curettage.
B) Pregnancy-related condition.
C) Sexually transmitted infections. Discussion 8-3
D) Anovulatory cycle. The correct answer is "D." Treatment of abnormal uterine
E) Bleeding disorder. bleeding depends on the severity of the bleeding, hemoglo­
bin level, and degree of associated hemodynamic changes.
Discussion 8-1 Mild cases associated with a hemoglobin level of 1 2 g/dL or
The correct answer is "A:' Most cases of abnormal vaginal higher require only reassurance, a multivitamin with iron,
bleeding in adolescence are secondary to dysfunctional uterine and close follow up. Individuals with a hemoglobin level of
bleeding from anovulatory cycles. Genital cancers are very rare 10 to 12 g/dL should take a combined oral contraceptive pill
in young adolescents. The differential diagnosis includes: (OCP) every 6 to 12 hours for 24 to 48 hours until the bleed­
• Pregnancy-related conditions (intrauterine or ectopic preg­ ing stops, taper to one pill per day, and then continue daily
nancy, spontaneous abortion, and molar-trophoblastic O CPs for 3 to 6 months. Patients with a hemoglobin level
disease) of less than 10 g/dL may require hospitalization and initial
• Infections (vaginitis, cervicitis, endometritis, salpingitis, and treatment of higher doses of estrogen if hemodynamically
pelvic inflammatory disease) unstable. This patient has anemia and is subsequently tachy­
cardic. She needs aggressive management.
• Other gynecologic conditions (ovarian cyst, genital cancers,
Question 8-4
breakthrough bleeding associated with contraceptive use,
ovulation bleeding, polyps, endometriosis)
The most common inherited bleeding disorder, which often
• Systemic disease (renal and liver failure) presents as menorrhagia in adolescent females is:
• Bleeding disorders A) Factor VIII deficiency.
• Direct trauma and foreign body B) Factor IX deficiency.
• Medications (anticoagulants and platelet inhibitors) C) von Willebrand disease.
D) Protein C deficiency.
E) Antiphospholipid syndrome.
Question 8-2
The following tests should be included in the workup of this Discussion 8-4
patient: The correct answer is "C:' The most common inherited bleeding
A) Pregnancy test. disorder in the United States population is von Willebrand dis­
B) Hemoglobin/hematocrit. ease, with an estimated prevalence of 1 % to 2%. The prevalence
C) Coagulation studies. of von Willebrand disease rises in studies involving women with
D) Nucleic acid amplification testing (NAAT) for gonorrhea menorrhagia, with estimates ranging as high as 10 % to 20% in
and chlamydia. Caucasian women, and 1 % to 2% among African American
E) All of the above. women. Von Willebrand factor helps with formation of the ini­
tial blood clot (binds platelets and factor VIII). Other bleeding
Discussion 8-2 disorders seen in adolescents with menorrhagia are disorders
The correct answer is "E:' A pregnancy test is essential to exclude of inherited platelet dysfunction, clotting factor deficiencies,
complications of pregnancy. Hemoglobin and hematocrit val­ thrombocytopenia, and disorders of the fibrinolytic pathway.
ues will help determine the magnitude of bleeding. Coagulation People with antiphospholipid syndrome and protein C and S
studies may reveal coagulopathies or blood dyscrasias, partic­ deficiencies are at increased risk for thromboembolism, not
ularly in this case of heavy bleeding since onset of menarche menorrhagia.
and a family history of abnormal vaginal bleeding. STis such
as gonorrhea and chlamydia are common causes of prolonged
or irregular vaginal bleeding. Other causes to consider include
endocrine abnormalities such as hypothyroidism.

The pregnancy test was negative. Her hemoglobin is 8.5 g/dL. A 16-year-old girl comes to your clinic requesting birth
control. She just started having sexual intercourse with her
Question 8-3 boyfriend of 1 year. They have used condoms with each epi­
What is the next appropriate step? sode of vaginal intercourse. Menarche was at 12 years of age.
A) Reassurance. Menses occurs every month, lasts for 5 days, and is associ­
B) Start a multivitamin with iron. ated with mild cramps. Her past medical history consists of
CHAPTER 1 • A D O L E S C E N T M E D I C I N E A N D GYN ECOLOGY 15

well-controlled absence seizures diagnosed at 16 years of age Question 9-2


for which she takes a medication. She had an appendectomy Combined OCPs can safely be used by an adolescent with any
in the past year; imaging during the diagnostic evaluation of the following conditions EXCEPT:
showed a left ovarian cyst measuring 1 .6 x 2 x 1 .8 em. Her A) Migraine with aura.
mother takes medication for high blood pressure and her B) Blood pressure of 1 3 5/80 mm Hg.
father has high cholesterol. C) Hypothyroidism.
D) History of ovarian torsion.
Question 9- 1 E) Asthma.
Which of the following contraceptive options has the lowest
failure rate during typical use in adolescence? Discussion 8-2
A) Male condom. The correct answer is "A:' According to the Centers for Disease
B) Combined OCPs. Control and Prevention's (CDC) US Medical Eligibility Criteria
C) Implantable etonogestrel rod. for Contraceptive Use, absolute (level 4) contraindications to
D) Injectable depo-medroxyprogesterone. combined OCP use include current breast cancer or estrogen­
E) Progestin-only pills. dependent tumor, thromboembolic disease or high risk for

Discussion 9- 1
thromboembolism (thrombogenic mutation, antiphospholipid

The correct answer is "C' See Table 1 - 5 for failure rates of dif­
antibody), migraine with aura, blood pressure of 1 60/ 1 00 mm
Hg or higher, cardiovascular disease, liver disease, and cerebro­
ferent methods of contraception. With so many choices, why vascular events.
aren't all sexually active adolescents using contraception and
STI protection? Perhaps the adolescent has difficultly planning
ahead, fails to recognize potential consequences, lives in the
moment, fears a pelvic exam or side effects, or has concerns over
confidentiality.
� QUICKQUIZ
Which is a noncontraceptive benefit of estrogen-progestin­
containing birth control?
TABLE 1-5 FA I LU R E RATES WITH P E R F ECT U S E
A) Increased risk of ovarian cancer.
A N D TYP I C A L U S E O F D I F F E R E N T CO NTRAC E PTIVE
B) Increased acne.
M ETHODS
C) Worsening menstrual cramps.
Failure Rate D) Decreased bone density.
E) Prevents menstrual migraines without aura.
With Perfect With Typical
Contraceptive Use (%) Use (%)
Discussion
Withd rawa l m ethod 4 22 The correct answer is "E". Combined hormone- contain­
Rhyt h m meth od/ 5 24 ing pills, patch, and vaginal ring improve bone density
period ic a bsti nence and decrease ( 1 ) risk of ovarian and endometrial cancer;
M a l e co ndom 2 18 (2) menstrual cycle disorders, including menorrhagia
and dysmenorrhea; ( 3 ) acne; and (4) p elvic pain from
Fem a l e co ndom 5 21
endometriosis.
Co m b i ned OCPs 0.3 9

Question 9-3
Co m b i ned tra n sder- 0.3 8
m a l patch
Which of the following antibiotics interferes with the contra­
Co m b i ned vag i n a l 0.3 8 ceptive effectiveness of the OCP?
ring A) Amoxicillin.
Progest i n -o n ly p i l l 0.3 9 B) Rifampin.
I njecta ble D M PA 0.2 6 C) Cephalosporin.
I m pl a nta ble 0.05 0.05
D) Sulfonamides.
etonog estre l rod
Levonorg estrei i U D 0.2 0.2 Discussion 9-3
The correct answer is "B:' Rifampin is the only antibiotic proven
Copper I U D 0.6 0.8
to decrease the contraceptive effectiveness of the pill. Amoxicil­
D M PA, d epot med roxyprogesterone acetate; I U D, i ntra u terine lin, cephalosporins, and sulfonamides do not affect birth con­
device; OCP, ora l contraceptive p i l l . trol effectiveness. Griseofulvin, an antifungal medication used
Data from Hatcher RA, Trusse l l J, N e l s o n AL, e t a l . Contraceptive to treat tinea infections, also speeds up metabolism of OCPs,
Technology. 20th rev ed. New York, NY: Ardent Media; 201 1 . decreasing their effectiveness.
16 MCG RAW-H I LL E D U CAT I O N S P E C I A LTY BOARD REVI EW: P E D I ATRICS

Question 9-4 classmate while at a party 3 months ago. She has not told any­
Which of the following antiseizure medications decreases the one about the assault and still sees the person who assaulted
contraceptive effectiveness of the OCP? her at school. She does not want to report the incidence
A) Gabapentin. because she doesn't want her boyfriend to know that he may
B) Phenobarbital. not be the father of her child.
C) Valproate.
Question 9-7
D) Levetiracetam.
E) All of the above.
Legally, you:
Discussion 9-4
A) Agree to her wishes to not disclose the assault.
B) Arrange for her boyfriend to undergo paternity testing.
The correct answer is "B:' Antiepileptics that induce the cyto­
C) Inform her that it is your legal obligation to report the
chrome P450 system (increasing metabolism), and therefore
assault to the authorities because she is a minor.
make the birth control pill less effective, include carbamazepine,
D) Inform her that you are going to tell her parents.
felbamate, phenobarbital, phenytoin, primidone, and topira­
mate. Lamotrigine clearance is increased in presence of estro­
gen-containing birth control options, meaning the effectiveness Discussion 9-7
of the seizure medication is decreased. The correct answer is "C:' All sexual assaults involving a minor
must be reported to authorities, even if the minor does not want
Your patient does not think she would remember to take a the assault disclosed.
birth control pill every day and would like to hear about the
other options.

Question 9-5
� QUICKQUlZ
Which of the following correctly matches the contraceptive
True or false: Pregnancy related risks in adolescents are age
with its duration of action?
dependent.
A) Transdermal patch; 3 weeks.
A) True.
B) Vaginal ring; 3 months.
B) False.
C) Levonorgestrel intrauterine device (IUD); 10 years.

Discussion
D) Copper IUD; 10 years.

The correct answer is ''A:' Younger adolescents are at increased


E) None of the above.

Discussion 8-5
risk for pregnancy complications, including poor weight gain,
anemia, pregnancy-induced hypertension, and poor prenatal
The correct answer is "D:' The birth control patch needs to be
care; are less likely to finish high school; and are more likely
changed weekly. The birth control ring is typically replaced after
to be single parents. They may lack the maturity to care for an
3 weeks. There are two different levonorgestrel IUDs; one lasts
infant. Infants of adolescent mothers are at increased risk for
for 3 years and the other for 5 years. The copper IUD is replaced
prematurity and low birth weight. Pregnant adolescents are
every 1 0 years.
more likely to live in poverty, not finish high school and require
governmental assistance. Care should be multidisciplinary,
She returns 6 months later and tells you that she is pregnant.
involve community resources, stress the importance of school,
Question 9-6
and provide positive reinforcement for successes.
Which of the following would put her at higher risk for an
ectopic pregnancy?
A) Prior episode of pelvic inflammatory disease.
B) History of OCP use.
C) History of labial adhesions requiring estrogen cream.
D) Frequent yeast infections. A 16-year-old girl presents to clinic with dysuria and vaginal
E) All of the above. discharge. She has had four sexual partners in her lifetime:
three male and one female. She started having sex at 12 years
Discussion 9-6 of age. The pH of the vaginal discharge obtained during
The correct answer is ''A:' Risk factors for ectopic pregnancies saline mount (wet prep) collection is 7.
include previous ectopic pregnancy; infection of the uterus,
fallopian tubes, or ovaries (pelvic inflammatory disease); preg­ Question 1 0-1
nancy when an IUD is in place; or pregnancy after tubal ligation. Which of the following is an unlikely cause of the vaginal
discharge?
She hasn't told anyone about her pregnancy. Upon further A) Bacterial vaginosis.
questioning, she discloses that she was sexually assaulted by a B) Trichomoniasis.
CHAPTER 1 • A D O L E S C E N T M E D I C I N E A N D GYN ECOLOGY 17

� QUICKQUlZ
C) Vaginal candidiasis.
D) Gardnerella vaginalis.
E) None of the above.
Which is NOT an indication for a pelvic exam?
Discussion 1 0-1 A) Amenorrhea.
The correct answer is "C:' The pH for yeast infection is less B) Pregnancy.
than 4.5 ( acidic) , and budding yeast or pseudohyphae can be C) Yearly checkup.
seen on KOH prep. The pH for bacterial vaginosis ( caused D) Persistent vaginal discharge.
by Gardnerella vagina/is) and trichomoniasis is greater than E) Dysuria in a sexually active female.

Discussion
4.5.

O n speculum exam, you see a red and friable cervix and The correct answer is "C". The guidelines for Papanicolaou
white, frothy discharge in the cul-de-sac. (Pap) testing have changed and noninvasive STI testing is avail­
able, so fewer adolescents need pelvic exams. (See Table 1 -6.)
Question 1 0-2 A Pap smear and pelvic exam are no longer required at the
On the wet mount, you are likely to find: onset of sexual activity or before prescribing birth control.
A) Trichomonads. Current guidelines recommend performing the first Pap test at
B) Budding yeast. age 2 1 years, with these exceptions: ( 1 ) immunocompromised
C) Pseudohyphae. (includes HIV) adolescents need annual tests once sexually
D) Clue cells. active, and (2) adolescents who have already been found to have
E) No-clue cells. cervical intraepithelial neoplasia ( CIN) 2 or 3 or carcinoma
need periodic screening and, in the case of CIN 3, treatment.
Discussion 1 0-2
The correct answer is "A:' The clinical appearance of the dis­
charge provides important clues to the diagnosis. Gray, frothy • Helpful Tip
discharge is consistent with trichomoniasis (along with the � Consider acute retrovira l synd rome i n a ny sexua l l y
appearance of the red, friable cervix, or "strawberry cer­ r1 1r active adol escent with nonspecific vira l sym ptoms,
vix") . The discharge associated with yeast infections is usually i n c l u d i n g fever, m a l a ise, lym phadenopathy, a n d skin
described as thick and curdlike, and adheres to vaginal walls. ras h . The synd rome occu rs i n the fi rst few weeks of
The discharge associated with BV is usually described as thin, i nfection, before a nti body testing is confi rmed as
grayish, and foul smelling. With BV, clue cells are present on positive. The test to order for acute retrovira l syn d rome
wet prep and the whiff test is positive. No one likes a fishy odor. is a n H I V PCR-D N A or H I V plasma RNA. Acutely infected
If you chose option "E;' it suggests that you were clueless about patients a re h i g h ly contagious d u ri ng t h i s stage.
this question.

Question 1 0-3 The patient does not want her parents to know that she is
What additional diagnostic tests should you order? being tested for STis.
A) Urine pregnancy test.
B) NAAT for gonorrhea.
C) NAAT for chlamydia.
TA B L E 1-6 I N D I CATI O N S FOR P E LV I C
D) HIV.
EXA M I NATION
E) All of the above.
Persi stent vag i n a l discharg e Sex u a l l y active with
Discussion 1 0-3 dys u ri a or u ri n a ry tract
The correct answer is "E:' The patient admits to having unpro­ sym pto m s
tected sex and already has been diagnosed with one STI ( tricho­ Dysmen orrhea n ot hel ped by Amenorrhea
monas) . She is at risk for other infections as well as pregnancy. NSAIDs
The CDC's STI screening guidelines for sexually active adoles­ Abnormal vag i n a l Lower abdom i n a l
cents include ( 1 ) annual Chlamydia trachomatis screening for bleed i n g pa i n
all females younger than 25 years old, (2) C. trachomatis screen­ I U D o r d i a p h ra g m contracep­ Perform P a p test
ing of males in certain settings, and (3) annual Neisseria gonor­ tive co u n se l i ng
rhoeae screening in all at-risk females. HIV screening should be
S u s pected or reported /ra pe Preg na ncy
discussed and encouraged for sexually active adolescents and
o r sex u a l a buse
those who use injection drugs. Screening for certain STis in
asymptomatic adolescents ( eg, syphilis, trichomoniasis, hepati­ I U D, i ntra uteri ne device; N S A I D, nonsteroidal a nti-infl a m matory
tis B) is not recommended. drug.
18 MCG RAW-H I LL E D U CAT I O N S P E C I A LTY BOARD REVI EW: P E D I ATRICS

Question 1 0-4 3 months after treatment. Reinfection rates are high, especially
You tell her: if partners are not treated.
A) Minors can consent to STI testing without parental
notification.
B) Minors require parent or guardian consent for STI testing. • Helpful Tip
=.� Empiric a ntibiotic prophylaxis after possible STI
i1 1 r exposu re incl udes ceftriaxone (250 mg intra m uscu l a r
C) Minors can consent to STI testing but parents must be
notified.
D) None of the above. i njection) for gonorrhea, azith romycin ( 1 g ora l ly) or
doxycyc l i n e ( 1 00 mg twice d a i l y for 7 days) for c h l a myd ia,

Discussion 1 0-4
and metron idazole (2 g ora l ly) for trichomoniasis.

The correct answer is ''A:' All 50 states and the District of


Columbia allow minors to consent to STI services without
The patient is seen in the emergency department 2 months
parental notification. Eighteen states allow, but do not require,
later with a 3-day history of fever and abdominal pain, but no
a physician to inform a minor's parents that he or she is seek­
ing or receiving STI services when the physician deems it in the dysuria, vomiting, or diarrhea. She states that a new vaginal
discharge developed 1 week ago. She continues to be sexually
minor's best interests. A key consideration is that you can't con­
active with sporadic use of condoms and has had one new male
trol what is on the insurance claims.
sexual partner. Her last episode of unprotected sex was 2 weeks
The patient wants to know what type of complications could ago. On exam, she is febrile to 38.5°C ( 1 0 1 .3°F) and has pain
occur if she was exposed to gonorrhea. and rebound tenderness in the left lower quadrant. On pelvic
exam, her SMR is 5 with normal external genitalia. On spec­

Question 1 0-5
ulum exam, there is discharge coming from the cervical os.
On bimanual exam, cervical motion tenderness and fullness
You explain that gonorrhea can cause all of the following
of the left adnexa are present.
EXCEPT:
Question 1 0-7
A) Rash.
B) Arthritis.
Which is NOT a diagnostic criterion for her acute condition?
C) Pelvic inflammatory disease.
A) Uterine tenderness.
D) Painless ulcer.
B) Adnexal tenderness.
E) Perihepatitis.
C) Lower abdominal pain.

Discussion 1 0-5
D) History of sexual activity.
E) Positive result for Neisseria gonorrhoeae.
The correct answer is "D:' Gonorrhea infection can cause muco­
purulent cervicitis, intermenstrual bleeding, and pelvic inflam­ Discussion 1 0-7
matory disease. Disseminated gonococcal infection occurs in The correct answer is "E:' Pelvic inflammatory disease (PID) is
1% to 3% of individuals with gonorrhea. It can present as pete­ an infection of the upper reproductive tract, including endo­
chial or pustular skin lesion, tenosynovitis, septic arthritis, and metritis, salpingitis, tubo-ovarian abscess, and pelvic peritoni­
perihepatitis (Fitz-Hugh-Curtis syndrome) . A painless ulcer is tis. Adolescent girls are disproportionately affected. Infection
indicative of primary syphilis (chancre), granuloma inguinale, occurs when lower genital tract (vagina, cervix) bacteria move
or lymphogranuloma venereum. into the upper genital organs. Symptoms range from mild to
severe. The CDC recommends treatment for presumed PID in a
The patient's boyfriend has a history of chlamydia, which was sexually active woman at risk for STis presenting with the mini­
picked up on routine screening. mum clinical criteria of ( 1 ) lower abdominal or pelvic pain and
(2) uterine, adnexal, or cervical motion tenderness. Supportive
Question 1 0-6 criteria include fever (> 38.3°C [ 1 00.9°F] ) , mucopurulent dis­
What is the treatment of chlamydial infections in men? charge (vaginal, cervix), white blood cells on saline microscopy,
A) Azithromycin. elevated inflammatory markers, and known positivity for N.
B) Ceftriaxone. gonorrhoeae or C. trachomatis. Don't forget you have to rule out
C) Penicillin. alternative causes first. (See Table 1 -7.)
D) Metronidazole.
E) Fluconazole. Question 1 0-8
Which is a bacterial cause of PID?
Discussion 1 0-6 A) Neisseria gonorrhea.
The correct answer is ''A:' The preferred treatment of chlamydia B) Chlamydia trachomatis.
consists of a single 1 g oral dose of azithromycin or a 1 -week C) Ureaplasma urealyticum.
course of doxycycline ( 1 00 mg twice daily) . Those who test D) Haemophilus influenzae.
positive for chlamydia or gonorrhea should have a test of cure E) All of these bacteria can cause PID.
CHAPTER 1 • A D O L E S C E N T M E D I C I N E A N D GYN ECOLOGY 19

TABLE 1-7 CAU S E S OF ACUTE P E LV I C PA I N


Discussion 1 0-1 0
The correct answer is "E". She has a tubo-ovarian abscess, which
U r i n a ry U ri n a ry tract i nfection can be diagnosed by ultrasound. All girls with PID need treat­
Pyeloneph ritis ment with broad-spectrum antibiotics. Hospitalization is indi­
N e p h ro l i t h i a s i s cated for surgical emergencies, pregnancy, failed outpatient
treatment, severe illness, or tubo-ovarian abscess. The latter
U ro l i t h i a s i s
may require surgical drainage.
Gastroi ntesti na I Appendicitis
Cholecystitis Her male sexual partner is contacted and he agrees to be
Mesenteric lymphaden itis evaluated. He reports burning with urination and penile dis­
charge for the last week. On exam, he is afebrile, SMR 5, and
I nfl a m matory bowe l d i sease
has erythema at the urethral meatus but no lesions. Purulent
Gastroenteritis discharge is seen after stripping the urethra.
Gynecologic Ova r i a n torsion, cyst, or tumor
Pelvic i n fl a m m atory d i sease Question 1 0-1 1
What do you tell him?
Ectopic preg n a n cy
A) He has an STI.
I ntrauterine preg n a n cy B) He does not require treatment.
Endometriosis C) He can continue to have sex as long as he wears a condom.
Hematometrocol pos D) He should avoid spermicides.
E) He does not need more testing.
M u sc u l os ke l eta l Psoas m u scle a bscess
Pelvic osteomye l itis Discussion 1 0-1 1
The correct answer is "!\: Urethritis is most commonly caused by
STis bacteria, including N. gonorrhoeae or C. trachomatis. Other
Discussion 1 0-8 causes include Ureaplasma urealyticum, Trichomonas vaginalis,
The correct answer is "E:' Multiple bacteria can cause PID, and herpes simplex virus. Noninfectious causes include trauma,
including STis, genital flora, respiratory pathogens, and enteric chemical irritation, Stevens-Johnson syndrome, Kawasaki disease,
organisms. PID is considered a polymicrobial infection and a and Reiter syndrome. Pyuria, white blood cells on Gram stain
causative agent is not always found. of a urethral swab, or mucopurulent discharge on exam confirm
the diagnosis. He should be tested for gonorrhea, chlamydia, and
Question 1 0-9 HIV If Gram stain of a urethral swab is suggestive of gonorrhea,
Which is NOT a complication of PID? he should be treated with ceftriaxone and azithromycin or doxy­
A) Tubo-ovarian abscess. cycline. If gonococci are not present, only azithromycin or doxycy­
B) Infertility. cline is needed. He should not have sex for 1 week after treatment
C) Renal abscess. to prevent transmission. Spermicides can cause chemical irritation,
D) Ectopic pregnancy. but he has an STI. Fill his pockets with condoms before he leaves.
E) Peritonitis.

Discussion 1 0-9
The correct answer is "C:' Short-term complications include
tubo-ovarian abscess, perihepatitis, and periappendicitis. Long­
An adolescent girl comes to the clinic for her yearly checkup.
term problems include infertility from tubal scarring, ectopic
Her major complaint is acne. She wants a cream to make it go
pregnancy, and chronic pelvic pain.
away. She reports feeling less attractive than her classmates.
She says that all the girls in her class get to go out on school
Her pregnancy test is negative. Saline mount (wet prep) reveals
nights but her mom won't let her, which means she will never
many white blood cells. KOH testing is negative. Endocervi­
be popular. When asked, she understands that her mother
cal specimens are sent for N. gonorrhea and C. trachomatis
doesn't want her to be too tired for school.
testing. Urinalysis and urine culture are normal.

Question 1 0-1 0 Question 1 1 - 1


What should be done next? Which period o f adolescence best describes this girl?
A) Order an abdominal ultrasound. A) Early adolescence.
B) Admit to the hospital. B) Middle adolescence.
C) Administer intravenous antibiotics. C) Late adolescence.
D) Consult a surgeon. D) None of the above.
E) All of the above. E) Adolescence has too many stages.
20 MCG RAW-H I LL E D U CAT I O N S P E C I A LTY BOARD REVI EW: P E D I ATRICS

TA B L E 1-8 C H A RACTE RISTICS O F EARLY, M I D D L E, AND LATE A D O L E S C E N C E

Early Adolescence Middle Adolescence Late Adolescence


( 1 1 - 1 4 years) ( 1 5 - 1 7 years) ( 1 8-2 1 years)
Physical G rowth s p u rt Pu berty co m pl eted Physica l ly mature
Seco ndary sex c h a racteristics G rowth s l ows G rowth co m p lete
Acn e and odor
Cog n itive Concrete t h i n ki n g Abstract th i n ki n g Futu re oriented
Poor problem solvi ng Stru g g l e s with decision U n dersta nds futu re
Lack i m p u lse control m a k i n g in times of stress conseq uences of deci s i o n s
Fee l i n g s of o m n i pote nce a n d Able t o co m p ro m i se a n d set
i nvi n c i b i l ity l i m its
Emotional Self-conscious about body i m age Self-centered Strong perso n a l ide ntity
Strug g les with sense of ide ntity Attractive ness co ncerns I ncreased emotional sta bi l ity
Mood swi ngs
I nte nse emotions
Socia l/Peers Same-sex re lati o n s h i ps Strong peer i nfl uence Less i n fl uenced by peers
Looks up to peers Seeks more i nti mate
Peers a re ro l e models relati o n s h i ps

Risk-ta king behaviors


Fa m i ly Des i res privacy Pea ked confl i ct with pa rents Accepts pare nta l advice
Less i nterest in fa m i ly activities
Sexua l Preocc u p i ed with changes i n Questio n s sexual orie ntation Esta b l i s h i n g sex u a l ide ntity
gen ita l s Dat i n g
Sexu a l activity

Discussion 1 1 - 1 D) Peers do not have a strong influence on likes, dislikes,


Th e correct answer i s "B:' Adolescence i s a time o f many choices, and behaviors.
changes and consists of three distinct periods: early, middle, E) She should take away her daughter's cell phone when she
and late. (See Table 1 -8.) The body is maturing and changing. "blows up:'
Rapid body changes may make an adolescent feel self-conscious
or awkward. Socially peers take on a larger role. Separation Discussion 1 1 -2
from family occurs. Thinking moves from concrete to abstract. The correct answer is "B." Emotional and physical separa­
The adolescent is determining who he or she is (self-identity) . tion from family is a normal part of adolescent development.
Middle adolescence is a time of acne, concerns over attractive­ Seeking autonomy, being argumentative, and demanding
ness, stereotypical behaviors, increased role of peers, the start of privacy are normal behaviors. Adolescents will spend more
abstract thinking and struggles with family for autonomy. time with peers. Peers or an adult outside the family (ie, coach)
will be sought for advice, emotional support, or both. Parents
The mother is frustrated with her daughter's attitude. They must adapt their parenting style based on the teen's devel­
are arguing more lately. What is especially trying is the ado­ opmental ability to think through problems, assess conse­
lescent will "blow up" then go to her room to be alone. Mom quences, and problem solve. Negotiation works better than
is worried that the girl is spending too much time texting being authoritative. Parents need to stay involved and sup­
friends on her cell phone. The mother wants to know if these portive as connectedness protects against risk-taking behav­
behaviors are normal. iors. Adolescents should be interviewed alone, allowing the
opportunity to talk freely. The discussion is confidential and
Question 1 1 -2 information should be shared with the parents only if it is
What do you tell her? life threatening. The AAP discourages drug testing without
A) Recommend drug testing without telling her daughter. consent except when the adolescent lacks decision-making
B) Spending more time with peers is normal. capacity or there is strong suspicion of drug abuse from the
C) Mood swings are uncommon during adolescence. history or exam. Issues surrounding minors and consent
CHAPTER 1 • A D O L E S C E N T M E D I C I N E A N D GYN ECOLOGY 21

are variable. If in doubt look it up on the Guttmacher Insti­ day before her cycle, lasting 2 to 3 days. She has missed 6 days
tute's website. In general, the adolescent should be given the of school in the past 6 months due to the pain. She has tried
opportunity to consent. acetaminophen at home with minimal relief. Menses started
at age 12, currently occur monthly, and last 5 to 6 days, with
moderate flow requiring her to change her pad/tampon every
� QUICKQUIZ 4 to 6 hours. She reports that she is attracted to boys and, when
asked confidentially, that she has never been sexually active.

Question 1 2- 1
Concrete thinking is characteristic of which stage of
adolescence?
What i s the most likely cause o f her abdominal pain?
A) Early.
A) Ovarian cyst.
B) Middle.
B) Endometriosis.
C) Late.
C) Primary dysmenorrhea.
Discussion
D) Pelvic infection.
The correct answer is ''A:' Concrete thinking is typically seen in
E) Endometrial polyp.

Discussion 1 2- 1
early adolescence (see Table 1-8). It is important to remember that
physical, cognitive, and social development may not be synchro­
Th e correct answer i s "C:' Dysmenorrhea o r pain with menses is
nous. For example, a 14-year-old patient who has almost completed
a common complaint in menstruating adolescents. Studies esti­
puberty would be classified as being in middle to late adolescence
mate that more than 50% of adolescents experience dysmenor­
based on secondary sex characteristics and physical development,
rhea. Primary dysmenorrhea is lower abdominal or pelvic pain
but may still have very concrete thinking and be classified as being
with menses that is not secondary to pelvic pathology. Pain is
in early adolescence in terms of cognitive development.
typically described as crampy, begins 1 to 2 days before men­
ses and may continue for 1 to 3 days into menses. Associated
• Helpful Tip symptoms include low back pain, nausea or vomiting, diarrhea,
� The CRAFFT screening tool for su bsta nce a buse is a useful fatigue, dizziness, and headaches. Dysmenorrhea symptoms
1 1 1 r tool for screening for high risk a lcohol and su bstance typically appear 1 to 3 years after menarche when regular ovula­
tory cycles have been established. The mechanism of primary
use d isorders in adolescents. The i nterviewer fi rst asks
an opening question asking if a patient has drunk a ny dysmenorrhea is prostaglandin formation from the endometrial
alcohol (more than a few si ps), smoked a ny ma rijua na, or lining with ovulatory cycles. The differential diagnosis for pri­
used a nyth ing else to get high. If the answer to all these mary dysmenorrhea includes all causes of secondary dysmenor­
questions is no, then only the first q uestion of the CRAFFT rhea. Gynecologic causes of pelvic pain include endometriosis,
screen is asked; if the answer is yes, then a l l six questions adenomyosis, fibroids, ovarian cysts, chronic PID, obstructive
a re asked. The screening tool consists of six questions: endometrial polyps, congenital obstructive mullerian malfor­
mations, cervical stenosis, and pelvic congestion syndrome.
C -Have you ever ridden in a CAR d riven by someone
Nongynecologic causes include inflammatory bowel disease,
( i n c l u d i n g you rself) who was "high" o r had been u s i n g
irritable bowel syndrome, and psychogenic disorders. Patients
a lcohol or d rugs?
with a typical history of dysmenorrhea as described above and
R -Do you ever use a lcohol o r d rugs to RELAX, feel
a normal pelvic exam can be diagnosed with primary dysmen­
better a bout you rself, or fit in? orrhea. Atypical elements of the history that may indicate an
A -Do you ever use a l cohol/d rugs while you a re by underlying cause of dysmenorrhea include pain that begins at
you rself, ALO N E? onset of menarche, which could be related to mullerian duct or
F -Do you ever FORG ET t h i n g s you d i d w h i l e u s i n g hymenal abnormalities; menstrual pain that becomes progres­
a lcohol or drugs? sively worse over time, which could be associated with endome­
F -Do you r fa m i l y o r F R I E N DS ever tel l you that you triosis; or a history of STis, which may be related to intrauterine
s h o u l d cut d own o n you r d r i n ki n g or d rug u se? or pelvic adhesions.

T -Have you g otten i nto TRO U B L E w h i l e you were


using a lcohol or drugs?
• Helpful Tip
:5.� Menses typica l ly beg i n between ages 1 2 a n d 1 3 yea rs i n
r1 1 r well-nourished American teens. Cycles can be irreg u l a r
i n the fi rst few years o f menses d u e t o a n ovu lation,
with typica l cycle i nterva l s ra n g i n g from 21 to 45 days.
A mother brings her 14-year-old daughter to your clinic with Length of menstrual flow is typica lly 2 to 7 days. Typica l
complaints of abdominal pain during her menstrual cycles. menstrual prod uct use is 3 to 6 pads or ta m pons a day.
The pain typically occurs in her lower abdomen and starts the
22 MCG RAW-H I LL E D U CAT I O N S P E C I A LTY BOARD REVI EW: P E D I ATRICS

Question 1 2-2 Bordini B , Rosenfield R . Normal pubertal development


What would be the most appropriate first line therapy for part II: Clinical aspects of puberty. Pediatr Rev.
management of these symptoms? 20 1 1 ;32(7) :28 1 -292.
A) Fluoxetine. Campbell MA, McGrath PJ. Use of medication by adolescents
B) Naproxen sodium. for the management of menstrual discomfort. Arch Pedi­
C) Tramadol. atr Adolesc Med. 1 997; 1 5 1 :905.
D) Antibiotics. Centers for Disease Control and Prevention. Sexually transmit­
E) Laparoscopy. ted disease guidelines, 20 1 0 . MMWR Morb Mortal Wkly
Rep. 2 0 1 0;59 (RR- 1 2 ) : 1 - 1 16 .
Centers for Disease Control and Prevention. US medical eligi­
Discussion 1 2-2 bility criteria for contraceptive use, 2 0 10 . MMWR Morb
The correct answer is "B:' Nonsteroidal anti-inflammatory drugs Mortal Wkly Rep. 2 0 1 0;59(RR-4) : 1 -88.
(NSAIDs) are considered first-line treatment for dysmenorrhea. Diamant AL, Schuster MA, McGuigan K, et al. Lesbians' sexual
Patients should be told to take the medication at the first sign of history with men: Implications for taking a sexual history.
cramps and continue to take it as scheduled through the first 1 to Arch Intern Med. 1 999; 1 59(22):2730-2736.
2 days of the cycle. Appropriate dosing should be reviewed with Dickey RP. Managing Contraceptive Pill Patients. 1 2th ed. New
the patient. Naproxen sodium is often preferable to ibuprofen Orleans, LA: Emis Medical Publishers; 2004.
due to ease of dosing. Tramadol can be used in patients who have Emans SJ. Amenorrhea in the adolescent. In: Emans SJ, Laufer
a contraindication to NSAIDs but is not typically considered a MR, Goldstein DP, eds. Pediatric & Adolescent Gynecol­
first -line agent. Exercise and other interventions to reduce stress ogy. 5th ed. Philadelphia, PA: Lippincott Williams &
can also be helpful. Adolescents should be evaluated within 3 to Wilkins; 2005:2 14-269.
4 months to evaluate the effectiveness of the medication. Emans SJ. Delayed puberty. In: Emans SJ, Laufer MR, Gold­
stein DP, eds. Pediatric & Adolescent Gynecology. 5th
The patient returns to the clinic 3 months later for follow up. ed. Philadelphia, PA: Lippincott Williams & Wilkins;
2005: 1 8 1 -2 1 3 .
Her pain is improved but continues to limit her activities 1
day a month. Emans SJ. Dysfunctional uterine bleeding. In: Emans SJ, Laufer
MR, Goldstein DP, eds. Pediatric & Adolescent Gynecol­
Question 1 2-3
ogy. 5th ed. Philadelphia, PA: Lippincott Williams &
Wilkins; 2005:270-286.
What is the most appropriate next step in management?
Pankowski B. Sexual orientation and adolescents. Pediatrics.
A) Referral to a gastroenterologist.
2004; 1 1 3 (6) : 1 827- 1 832.
B) Laparoscopy.
Fleishman A, Gordon CM, Neinstein LS. Menstrual dis­
C) Combination OCPs.
orders: Amenorrhea and polycystic ovary syndrome.
D) Pelvic ultrasound. In: Neinstein LS, ed. Handbook of Adolescent Health­
care. Philadelphia, PA: Lippincott Williams & Wilkins;
Discussion 1 2-3 2009:470-478.
The correct answer is "C:' If a patient continues to have symp­ Garofalo R, Katz E. Health care issues of gay and lesbian youth.
toms after a trial of NSAIDs, or if a patient needs contraception, Curr Opin Pediatr. 200 1 ; 1 3 (4):298-302.
combination OCPs can be started. OCPs work by suppressing Garofalo R, Wolf RC, Kessel S, et al. The association between
ovulation, which leads to a thinner endometrial lining with health risk behaviors and sexual orientation among
decreased prostaglandin production and bleeding. If symptoms a school-based sample of adolescents. Pediatrics.
continue after 3 to 4 months on OCPs, laparoscopy and a pelvic 1 998; 1 0 1 (5) :895-902.
ultrasound should be considered to evaluate for causes of sec­ Gray SH. Menstrual disorders. Pediatr Rev. 20 1 3;34( 1 ) :6- 1 8 .
ondary dysmenorrhea. Guttmacher Institute. State policies i n brief: Minors' access to
STI services, as of October 1 , 20 1 5 . Accessed October 2,
20 1 5 from http:/ /www.guttmacher.org/ statecenterI spibs/
B I B LIOGRAPHY Hatcher RA, Trussell J, Nelson AL, et al. Contraceptive Technol-
ogy. 20th rev ed. New York NY: Ardent Media; 20 1 1 .
American Academy of Pediatrics, Committee on Adolescence
and Committee on Early Childhood and Adoption, Hwang LY, Shafer M . Vaginitis and vaginosis. In: Neinstein LS,
and Dependent Care. American Academy of Pediatrics: ed. Handbook of Adolescent Healthcare. Philadelphia, PA:
Care of adolescent parents and their children. Pediatrics. Lippincott Williams & Wilkins; 2009:49 1 -498.
200 1 ; 1 07(2) :429-434. Joffe A: Gynecomastia. In: Neinstein LS, ed. Handbook ofAdo­
American Academy of Pediatrics Committee on Substance lescent Healthcare. Philadelphia, PA: Lippincott Williams
Abuse. Testing for drugs of abuse in children and adoles­ & Wilkins; 2009: 1 08- 1 1 1 .

cents. Pediatrics. 1 996;98:305-307. Kaplowitz PB. Delayed puberty. Pediatr Rev.


American Psychiatric Association. Diagnostic and Statistical 2 0 1 0;3 1 (5):5 1 89-5 195.
Manual of Mental Disorders. 5th ed. Washington, DC: Klein JR, Litt IF: Epidemiology of adolescent dysmenorrhea.
American Psychiatric Association; 20 1 3 . Pediatrics. 1 98 1 ;68:66 1 .
CHAPTER 1 • A D O L E S C E N T M E D I C I N E A N D GYN ECOLOGY 23

Levy S, Siqueira LM. Testing for drugs of abuse in children and Rodeghiero F, Castaman G, Dini E. Epidemiological investiga­
adolescents. Pediatrics. 2014; 1 3 3 (6):e 1 798- 1 807. tion of the prevalence of von Willebrand's disease. Blood.
Marrazzo JM, Koutsky LA, Kiviat NB, et al. Papanicolaou test 1 987;69(2) :454-459.
screening and prevalence of genital human papilloma­ Rosen DS. Identification and management of eating dis­
virus among women who have sex with women. Am J orders in children and adolescents. Pediatrics.
Public Health. 200 1 ; 9 1 (6) :947-952. 20 1 0 ; 1 26 ( 6 ) : 1 240- 1253.
Master-Hunter T, Heiman DL. Amenorrhea: Evaluation and Rosenfield RL, Lipton RB, Drum ML. Thelarche, pubarche,
treatment. Am Pam Physician. 2006;73 (8) : 1 3 74- 1 382. and menarche attainment in children with normal and
Nicoletti AM. Teen pregnancy. In: Emans SJ, Laufer MR, elevated body mass index. Pediatrics. 2009; 123:84-88
Goldstein DP, eds. Pediatric & Adolescent Gynecology. Trent M. Pelvic inflammatory disease. Pediatr Rev.
5th ed. Philadelphia, PA: Lippincott Williams & Wilkins; 2 0 1 3;34(4) : 1 63- 1 72.
2005:844-878. Wilson CA, Keye WR: A survey of adolescent dysmenorrhea
Reddy DS. Clinical pharmacokinetic interactions between and premenstrual symptom frequency. A model program
antiepileptic drugs and hormonal contraceptives. Expert for prevention, detection, and treatment. J Adolesc Health
Rev Clin Pharmacal. 20 1 0;3 (2) : 1 83 - 1 92. Care. 1 989; 1 0 : 3 1 7 .
This page intentionally left blank
Allergic and I m munologic Disorders 2
Amy 0. T h o m a s a n d A l ex Th o m a s

Question 1 -2
What should be the next step in confirming diagnosis?
A newborn boy i s delivered at 38 weeks' gestation after an A) Arrange for flow cytometry to enumerate B-, T-, and NK-
uncomplicated pregnancy and delivery. He is discharged cell populations.
home from the hospital on day of life 2. You are contacted by B) Obtain an echocardiogram.
the lab after it was noted that the newborn screen revealed a C) Obtain quantitative immunoglobulins.
T-cell receptor excision circle (TREC) of 0. D) Watch and wait for development of any clinical signs of
infection.
Question 1 - 1
What diagnosis should you be concerned of with this finding? Discussion 1 -2
A) Severe combined immunodeficiency (SCID). The correct answer is "A:' SCID is defined by severely diminished
B) Complete DiGeorge syndrome. T-cell function and abnormal B-cell function. It affects between
C) Bruton's X-linked agammaglobulinemia (XLA). 1 in 50,000 and 1 in 70,000 births. SCID is a medical emergency
D) Transient hypogammaglobulinemia of infancy (THI). and requires careful but prompt diagnosis once suspected. Flow
E) A and B only. cytometry looks at cell counts identified by cell surface markers.
F) All of the above. Both SCID and DiGeorge syndrome results in deficient T-cell
development. DiGeorge syndrome develops from a mutation
Discussion 1 - 1 on the long arm of chromosome 22, and patients often have
The correct answer is "E:' TRECs serve as a biomarker for nai:ve characteristic morphologic findings, cardiac abnormalities, and
T-cell production. They are found in 70% of nai:ve T cells. They hypoparathyroidism. In patients with complete DiGeorge, the
have served as a rapid DNA-based newborn screening tool immune dysfunction results in absent T cells and nonfunction­
for disorders of the immune system manifesting with T-cell ing B cells, with consequences similar to SCID. Thymic trans­
lymphopenia, such as cases of SCID, ataxia-telangiectasia, or plant is curative. For partial DiGeorge, as the name implies, the
complete DiGeorge syndrome with athymia. T-cell lympho­ immune defect is less severe, with some functional T- and B-cell
penia has also been noted in some cases of trisomy 2 1 and capabilities that often improve over time. Do you remember or
preterm birth with very low birth weight ( < 800 g), but counts have you heard of the boy in the bubble? A boy from Texas with
should not be completely absent. XLA will manifest with X-linked SCID lived for 12 years in a plastic, germ-free bubble.
absent mature B cells due to arrest in the pre- B-cell stage of His story brought national attention to SCID.
development. This will show with absent circulating immuno­
globulins of all classes. THI is due to prolongation of the nor­
• Helpful Tip
mal physiologic nadir of immunoglobulin G (IgG) production
=.� A low lymphocyte count is a tipoff to a T-cel l problem as
that occurs in most infants between 4 and 6 months of age,
following a drop in circulating maternal IgG. Typically, this
r1 1 r the majority of circulating lymphocytes (60% to 70%) are
T cells. Flow cytometry can identify more specifically the
is outgrown by 3 to 4 years of age, with few children requir­
subpopulations using cell surface markers of B (CD1 9,
ing immunoglobulin replacement. IgM is typically normal in
CD20), T (CD3CD4, CD3CD8), and NK (CD1 6, CDS6) cell lines.
these infants.

25
26 MCG RAW-H I LL E D U CAT I O N S P E C I A LTY BOARD REVI EW: P E D I ATRICS

NK cells, but normal circulating numbers of B cells. Alarmed,


you send off further immune studies, including quantitative
You are starting in a new pediatric practice in the rural out­ immunoglobulins, which are found to be universally low.
skirts of a small town in Pennsylvania. Your first patient Mitogen proliferation studies show an absent response to
of the day is an 1 8-month-old boy who is presenting with phytohemagglutinin, pokeweed mitogen, and candida.
a 4-week history of a persistent dry cough now progressing
to dyspnea. On review of the history with parents, they state Question 2-2
he was born full term without issue, via a home delivery. What is the likely defect causing his condition?
He was treated for four episodes of otitis media in his first A) Adenosine deaminase (ADA) deficiency.
year of life, most of which required back-to-hack courses B) Common gamma chain mutation.
of antibiotics to clear. At 9 months he was hospitalized for C) Recombination-activating gene 1 ( RA G 1 ) deficiency.
pneumonia and dehydration secondary to diarrhea. As you D) gp9 1 phox mutation in the nicotinamide adenine dinucleo­
piece through the remainder of his chart, you note there has tide phosphate (NADPH) oxidase complex.
been no weight gain for the past 4 months and he has now
dropped several percentiles on his growth chart. Currently, Discussion 2-2
he is tachypneic with rales and rhonchi noted. He is slightly The correct answer is "B:' X-linked SCID is the most common
tachycardic with a temperature of 37.3°C (99.2°F) . You note cause of SCID (�50% of cases) and is caused by a defect in the
the oral cavity has what appears to be several white patches common gamma chain, a key protein involved in the signal­
with surrounding erythema and nearly absent tonsils. His ing cascade used in lymphocyte maturation and proliferation.
oxygen saturation is 88% on room air in the office. With X-linked recessive inheritance, only males are affected.
The common gamma chain is shared by several key signal­
Question 2-1 ing receptors and pathways including interleukin-2 receptor
In addition to acute management of this patient, what diag­ (IL2R) , IL4R, IL7R, IL9R, IL 1 5R, and IL2 1 R and the associ­
nosis must you consider and address urgently? ated downstream Janus kinase 3 (JAK3) signaling protein. In
A) Selective antibody deficiency. X-linked SCID, T and NK cells are typically low or absent, but
B) Enlarged adenoids. B cell numbers can be normal (T-B+NK- SCID ). However,
C) SCID. B cells cannot properly function without help from T cells, so
D) None of the above; this is a normal infection history for a even if B cells are not directly affected by a mutation, they will
child his age. be rendered nonfunctional. Deficiency of JAK3 due to a muta­
tion in the JAK3 gene will also cause T-B+NK- SCID but is
Discussion 2-1 inherited in an autosomal recessive mode. In the case of your
The correct answer is "C:' Look for the history of immune defi­ 1 8-month-old patient, both the flow cytometry profile as well
ciency that may have escaped detection on a newborn screen as family history of normal females but affected males on the
(see Case 1 ) . In the current case, the history of a "home birth" maternal lineage suggests X-linked SCID as the diagnosis.
should be followed with questions of whether a newborn screen Among autosomal recessive forms of SCID, ADA deficiency
was submitted. SCID will most often present in the first few (leading to toxic buildup of metabolites in T and B cells) and
months of life with chronic lung infections, diarrhea, thrush, RAG mutations (leading to impaired somatic gene recombina­
and failure to thrive. With T- and B-cell deficiencies, lymphoid tion necessary for B- and T-cell receptor synthesis) are among
tissue (lymph nodes, tonsils, adenoids, thymus) may be absent the most common. Both of these mutations will lead to absent
or reduced in size. Thrush outside of the infancy period is B and T cells (T-B-NK- SCID) on flow analysis. Chronic granu­
atypical and should not be dismissed as normal. Possible causes lomatous disease is associated with errors in the assembly of
include a primary or secondary cellular immunodeficiency, use the NADPH oxidase complex, of which gp9 l phox is a major
of inhaled corticosteroids, chemotherapy, and antibiotics. protein. B and T cells are not directly affected by this mutation.
(See Table 2- 1 .)
He is admitted to the hospital for further care. On arrival, a
chest x-ray reveals bilateral pneumonia and an absent thymic
shadow, prompting the start of empiric antibiotics and further • Helpful Tip
immunology evaluation. A sputum culture reveals respira­ =.� I n a n i m m u nosu ppressed patient presenti ng with
tory syncytial virus and adenovirus. A peripheral CBC shows r1 1 r history of persistent cou g h or dyspnea, concern m u st be
a WBC of 4000 cells/!!L, with noted lymphopenia (absolute raised for i nfection with opportu nistic i nfections such
lymphocyte count of 400 cells/!!L) . A quick glance at the fam­ as Pneumocystis jiroveci (formerly Pneumocystis carinil).
ily history shows he has a 5-year-old sister and an 8-year-old In i m m u nocom promised patients, look for a history
sister, both of whom are healthy. A maternal uncle died in early of atypical or severe i nfections such as pneumocystis,
childhood of overwhelming infection, but there is little other cytomega lovirus, rotavirus, mycobacteria, and
available information about his cause of death. The results asperg i l l us.
of flow cytometry return, and you note absent T cells, absent
CHAPTER 2 • A L L E RG I C AND I M M U N O LOG I C D I SO R D E R S 27

TABLE 2-1 SEVERE COM B I N E D I M M U N O D E F I C I E NCY


(SCID) P H E N OTYPES
� QUICKQUIZ
Abnormality SC I D Mutation
Which is not a potential clinical indicator of an immuno­
T- B- N K­ ADA deficiency deficiency?
T- B- N K+ RAG, O m e n n syn d rome A) Measles after receiving the MMR vaccine.
B) Recurrent thrush.
T- B + N K­ Co m m o n gamma c h a i n , JAK3, P N P
C) Chronic diarrhea.
T- B + N K+ CD3, I L7 R D) Liver abscess.
T+ B + N K+ Zap70 E) Recurrent self-limited viral upper respiratory tract infec­
tions in a child in daycare.
ADA, adenosine d ea m i nase deficiency; PNP, purine n u c l eoside
phosphorylase; RAG, reco m binant activating genes; S C I D, severe
Discussion
com b i n ed i m m u nodeficiency.
The correct answer is "E:' The average child can have several
"illnesses" in the span of a year without concern for underlying
immune deficiency. In fact, infants and children may have 8 to

� • Helpful Tip
10 respiratory infections in a year with symptoms lasting at least
SCID patients a re a l most always lym phopenic; however,
1 week per infection. However, a history of recurrent deep-seated

I I reca l l that age-adj u sted normal ra nges a re ava i l a ble for infections or abscesses, antibiotic usage with little effect or require­
c h i l d ren. Normal infa nt lymphocyte counts a re genera l ly ment for multiple back to back courses, and failure to thrive or loss
h igher than those for older c h i l d ren a n d a d u lts. of linear growth should all prompt further workup for primary
immune deficiency. In addition to recurrent infection, patients
with immune deficiency are at increased risk for development

Question 2-3
of autoimmune disease and malignancy. (See Table 2-2.) Signs
of a cellular immunodeficiency include recurrent viral, fungal,
What additional steps should be taken in his care?
or mycobacterium infections; chronic diarrhea; failure to thrive;
A) Start intravenous immunoglobulin (IVIG) for immuno­
and reaction to live viral vaccines. Symptoms may present early
globulin replacement.
B) Place him on prophylactic antifungal and antiviral medica-
tions to prevent the development ofopportunistic infections. TABLE 2-2 RED F LAGS FOR AN I M M U N O D E F I C I E NCY
C) Begin HLA typing for bone marrow transplantation.
D) Avoid live viral vaccines and nonirradiated blood products. Fa m i ly h i story of an i m m u nodeficiency
E) All of the above. Fa i l u re to th rive
C h ro n i c d i a rrhea
Discussion 2-3
I nfections req u i ri n g i ntraven ou s a nt ib iotics
The correct answer is "E:' B cells require T cells to assist in normal
Seve re or l ife-th reate n i n g i nfecti o n s
development and production of functional antibodies. There­
fore this patient will need supplemental IVIG to supplement his Rec u rre nt a bscesses, espec i a l ly t h o s e i nvolv i n g solid
humoral response and prophylactic antifungals and antivirals to o rg a n s (eg, l iver)
protect against opportunistic infections from the deficient cellu­ Rec u rre nt s i n u sitis, p n e u m o n ia, or otitis media
lar immunity. Early bone marrow transplantation is the preferred I nfection with opportu n i stic or atypical orga n i s m s (eg,
method of treatment for SCID, ideally before any major infections Burkho/deria, Pneumocystis jiroveci)
have occurred. While bone marrow transplantation will restore
Rec u r rent ca n d i d i a s i s
T-cell function, B-cell function typically remains abnormal, and
thus IVIG must often be continued indefinitely. Attenuated live Poor wou n d h ea l i ng
virus vaccines must be avoided, and any blood products used G ra n u lomas
must be irradiated. Nonirradiated blood products contain T cells Lym p h o m a i n i nfa ncy
which could lead to development of graft-versus-host disease.
Co m p l icatio n s fro m l i ve viral vacci nes (vacc i n e-associ­
ated i nfection)
Lym p hope n i a
• Helpful Tip
� O m e n n synd rome (a form of SCI D) is a va riant of RAG Eos i n o p h i l i a

r1 1r deficiency (pa rtial ina ctivation), m a n ifesting with Vacc i n e u n responsiveness


eryt h roderma, hepatospl enomega ly, chronic d i a rrhea, Del ayed hyperse nsitivity s k i n res ponses (low sensitivity)
fa i l u re to th rive, eosi noph i l ia, and elevated lgE levels. R h e u m atolog ic d i sorders
28 MCG RAW-H I LL E D U CAT I O N S P E C I A LTY BOARD REVI EW: P E D I ATRICS

in infancy. Signs of a humoral immunodeficiency include recur­ Complement deficiency can be due to a deficiency in the alternative
rent bacterial infections (eg, sinopulmonary, otitis), infections or classical pathway. Overwhelming Neisseria sepsis is a clue to an
with encapsulated bacteria, and enterovirus infections. Humoral underlying complement deficiency. Most of these are inherited in an
immunodeficiencies present later in infancy (> 4 months) or autosomal recessive fashion. Classically patients with complement
beyond due to maternal transplacentally acquired IgG. deficiencies present with severe or recurrent Neisseria infections
such as meningococcemia or meningococcal meningitis.

A 3-year-old boy presents to your office with his sixth ear infec­
tion this year. He has concurrent purulent nasal drainage and
his parents feel he is "always congested" despite being on anti­ An 18-year-old boy presents to the emergency department with
biotics as recently as 2 weeks ago. He was treated for pneumo­ abdominal pain, shortness of breath, and cough. He reports that
nia confirmed on chest X-ray at 21 months of age. Last year, symptoms began approximately 3 weeks ago but have progres­
he developed osteomyelitis of the tibia after a fall at the play­ sively worsened in the past 48 hours. He notes that he has been
ground, requiring a 4-week course of intravenous antibiotics. treated for pneumonia once or twice per year for the last 5 years.
He has had few to no issues with viral or fungal infections. He He has several scarred areas on his arms where he states previ­
is growing and developing normally and has had no issues with ous skin infections "healed funny." A poorly healing ulcer is noted
excessive bleeding or bruising. There is no history of recurrent on his lower leg that he said developed after being scraped while
rashes. On close examination, you note the absence of tonsils. helping to clear shrubbery on his father's farm. Chest X-ray shows
"cotton ball densities" in both lungs. A fine needle aspiration
Question 3-1
of the lung lesions is obtained and shows Aspergillus fumigatus.
Abdominal CT shows a multicentric liver abscess that contains
In addition to treating the current infection, what diagnosis
Serratia marcescens. IV antibiotics and antifungal medications are
should you consider?
started and he is admitted to the hospital for further care. He has a
A) None; this is a typical infection pattern for a child of his age.
mildly elevated WBC of 14,000 with slight predominance of neu­
B) Bruton's agammaglobulinemia.
trophils (75%). Quantitative immunoglobulin levels are normal.
C) SCID.
Question 4- 1
D) Complement deficiency.
E) Wiskott-Aldrich syndrome.
Based on his presentation and infection history, what is the
most likely underlying diagnosis?
Discussion 3 - 1 A) Common variable immunodeficiency (CVID).
The correct answer is "B:' X-linked agammaglobulinemia (XLA), B) Severe combined immunodeficiency ( SCID).
also known as Bruton's agammaglobulinemia, occurs due to a muta­ C) Kostmann syndrome.
tion in the Bruton tyrosine kinase (btk) gene (responsible for a tyro­ D) Chronic granulomatous disease (CGD).
sine kinase protein), causing arrest of B cells in the pre-B-cell stage.
Prevention ofB-cell maturation leads to absence of plasma cells and Discussion 4-1
the inability to produce circulating immunoglobulins. Lymphoid tis­ The correct answer is "D:' CGD is characterized by defective
sues where B cells are often found concentrated, such as the tonsils, intracellular killing of bacterial and fungal infections by phago­
adenoids, spleen, and lymph nodes, are often atrophic on physical cytes (neutrophils, macrophages, monocytes) . Commonly,
exam. The T-cell count is paradoxically often noted to be elevated. CGD patients have infections with catalase positive organisms
The absence of humoral immunity leads to increased infection with such as Staphylococcus aureus, S. marcescens, and Burkholderia
encapsulated organisms such as Streptococcus pneumoniae, Pseu­ cepacia complex, as well as fungal organisms such as Aspergil­
domonas species, or Haemophilus influenzae type B. Enterovirus is lus. The disease results from a mutation in the NADPH oxi­
also common in humoral deficiency. As implied in the name, it is dase complex, leading to failed production of superoxide by
inherited in an X-linked recessive fashion, thus only affecting males. the phagosome (respiratory burst) and ineffective killing and
Female carriers are asymptomatic. Diagnosis is made with screen­ clearance of intracellular organisms. Neutrophil numbers can
ing for low or absent immunoglobulin levels (IgG, IgM, IgA). Con­ be normal or elevated; however, their function is impaired.
firmation is made through flow cytometry with noted absent B-cell Severe congenital neutropenia (Kostmann syndrome) is due to
profile (look for low markers of CD 19, CD20). Treatment is with a mutation in the HAXl gene leading to chronic neutropenia
immunoglobulin replacement (IVIG). SCID would place a child at (not seen in this patient) . CVID is the most common primary
increased risk of fungal and viral infections with impaired cellular immunodeficiency. B cells fail to fully differentiate into plasma
immunity. Wiskott-Aldrich syndrome (WAS) is an X-linked reces­ cells, leading to a deficiency in some (IgG, IgM, IgA) or all of the
sive disease with characteristic severe eczema, thrombocytopenia, immunoglobulin subtypes as well as poor vaccine responses.
and immunodeficiency (cellular and humoral) due to mutations There is an increased risk of autoimmunity and malignancy
in the WAS gene which encodes for the Wiskott-Aldrich syn­ in patients with CVID. IVIG is required for long-term immu­
drome protein (WASp). Small platelets are noted on blood smear. noglobulin replacement. For diagnosis, look for absent or
CHAPTER 2 • A L L E RG I C A N D I M M U N O LOG I C D I SO R D E R S 29

low levels of class-switched memory B cells in CVID patients Discussion 4-4


(IgM-IgD-CD27+CD 1 9 + ) . The correct answer is "E:' Common infections in CGD include
pneumonia, lymphadenitis, skin abscess, liver abscess, perianal
Question 4-2 abscess, and osteomyelitis. Staphylococcal liver abscesses are
What test should be ordered to confirm your suspicion? strongly associated with CGD. Additionally, granulomas (col­
A) Vaccine titers and mitogen proliferation studies. lection of inflammatory cells) are common, which may obstruct
B) CD l l , CD 1 8 markers on flow cytometry. the urinary or gastrointestinal tract. Some patients develop
C) Dihydrorhodamine 12 3 fluorescence assay. Crohn-like inflammatory bowel disease.
D) HIV viral load, as an antibody test would be unreliable in
this patient.

Discussion 4-2 � QUICKQUIZ


The correct answer is "C:' Diagnosis of suspected CGD is made
through use of dihydrorhodamine (DHR) assay. DHR is a non­ A 3-year-old boy has had three episodes of cellulitis and two
fluorescent dye used to stain whole blood phagocytes, which are episodes of pneumonia in the last year. Both were caused
then stimulated with phorbol myristate acetate (PMA) . Normal by Staphylococcus aureus. His gums bleed chronically, espe­
phagocytes are able to oxidize the dye, causing increased fluo­ cially with brushing his teeth. On exam, he has mouth ulcers,
rescence, through production of superoxide. The fraction of inflamed gingiva with areas of bleeding, and a few nonheal­
fluorescent cells is measured by use of a fluorescence-activated ing wounds on his legs and arms.
cell-sorting machine (FACS) . Older techniques employed used
of nitroblue tetrazolium (NBT) , a dye that turned from yellow Which of the following immunodeficiencies is included in
to blue when reduced by superoxide. DHR is thought to be a the differential diagnosis for this child?
more accurate measurement and allows for distinction between A) Leukocyte adhesion deficiency.
X -linked recessive and autosomal recessive forms of the disease. B) Chediak-Higashi syndrome.

Question 4-3
C) Hyperimmunoglobulin E syndrome.
D) Chronic granulomatous disease.
In addition to the identified pathogens, which other patho­ E) All of above are possible diagnoses.
gen is commonly associated with the type of immunodefi­
ciency seen in this patient?
A) Salmonella typhimurium. Discussion
B) Chromobacterium violaceum. The correct answer is "E:' This case screams that the neu­
C) Actinomyces species. trophils are not working or not present. Phagocytes include
D) Nocardia species. neutrophils, monocytes, and macrophages and are part of
E) All of the above. the innate immune system. Primary phagocytic disorders
present with recurrent bacterial ( staphylococci) and fungal
Discussion 4-3 (aspergilli, candida) infections of the respiratory tract and
The correct answer is "E:' skin. Mucositis (mouth ulcers) , gingivitis, and poor wound
healing may also occur. Neutropenia has multiple causes that
may be congenital or acquired. Chediak-Higashi syndrome,
• Helpful Tip
an autosomal recessive condition, is associated with skin and
=.� I nfections with cata lase-positive bacteria a n d fu n g i a re
r1 1 r c haracteristic of CGD. The most common pathogen is
eye albinism, mild coagulopathy, neutropenia, and neuro­
logic problems (nystagmus, ataxia, neuropathies ) . Charac­
Staphylococcus aureus. teristic large azurophil granules are seen in neutrophils and
other granulocytes. Leukocyte adhesion deficiencies result
in the inability of neutrophils to undergo chemotaxis from
You tell the patient he has an immunodeficiency disease, the bloodstream to the site of infection; therefore, pus is not
explaining that certain cells in his body are unable to kill bac­ made and blood neutrophil counts are elevated. With infec­
teria and fungi. He wants to know what type of infections he tion, neutrophil counts may reach 1 00,000 cells/mm3• A
is susceptible to as a result of his disease. classic clue is delayed separation of the umbilical cord ( > 1
month) . Autosomal dominant hyper-IgE syndrome (HIES ) ,
Question 4-4 as known a s Job syndrome, is characterized b y recurrent
Which of the following is associated with CGD? abscesses from S. aureus, coarse facies, and severe eczema­
A) Pneumonia. like rash. Abscesses may be "cold" or not appear inflamed.
B) Colitis. Serum IgE levels are elevated and eosinophilia is present.
C) Liver abscess. Pneumonia may lead to abscess and pneumatocele forma­
D) Recurrent skin abscesses. tion. Other characteristics include retained primary teeth
E) All of the above. and skeletal fractures from minor trauma.
30 MCG RAW-H I LL E D U CAT I O N S P E C I A LTY BOARD REVI EW: P E D I ATRICS

• Helpful Tip
=� Live vira l vaccines s h o u l d be avoided i n patients with
An 8-year-old boy is new to your practice. You note as he
enters the room with his mother that he seems somewhat
i1 1r cel l u l a r (T-cell) i m m u n odeficiencies. If g iven, the virus
may cause i nfection i n the patient.
"off balance" to which his mom replies that she had noted
changes in his gait for the last 3 to 4 years, but thought he
was just "clumsy:' You are reviewing his medical history and
find that he has been treated for recurrent otitis media and
� QUICKQUlZ
sinusitis despite having three sets of tympanostomy tubes
since age 2. On examination, you note several superficial Which immunodeficiency is not correctly matched with its
blood vessels apparent in the eyes and on the pinna of the mode of inheritance?
ear. Concerned with the history and examination findings, A) Chronic granulomatous disease-X-linked recessive and
you order a CBC that demonstrates profound lymphopenia autosomal recessive.
as well as quantitative immunoglobulin levels and vaccine B) Ataxia-telangiectasia (AT)-autosomal dominant.
titers to pneumococcus and tetanus, all of which are low. C) X-linked agammaglobulinemia-X-linked recessive.
D) Severe combined immunodeficiency (SCID) -X-linked
Question 5-1 recessive and autosomal recessive.
What diagnosis are you most concerned with based on the E) Wiskott-Aldrich syndrome-X-linked recessive.
constellation of symptoms?
Discussion
A) Neuroblastoma.
The correct answer is "B:' AT is an autosomal recessive condition.
B) Chediak-Higashi syndrome.
C) WHIM syndrome.
D) Ataxia-telangiectasia.
E) Hyper-IgM syndrome.
An 1 8-year-old girl injured her knee after waterskiing at her
Discussion 5-1
family's lake house. She saw her primary care physician who
The correct answer is "D:' Ataxia-telangiectasia (AT) is caused by advised rest and icing, as well as ibuprofen if needed for pain.
a defect in ATM repair gene, which leads to a combined B- and Thirty minutes after taking 400 mg of ibuprofen, she develops
T-cell defect. Low immunoglobulin levels and lymphopenia will diffuse hives and swelling of her lips and eyelids. Alarmed,
be noted on screening labs. Progressive cerebellar ataxia and neu­ her family rushes her to the emergency department, where on
ron loss occur with age. Ataxia is the earliest clinical manifesta­ arrival she vomits, feels short of breath, and is lightheaded.
tion, with initial symptoms beginning around 2 to 3 years of age.
Telangiectasias of the bulbar conjunctiva and skin typically man­ Question 6- 1
ifest with disease progression. AT is inherited in an autosomal What medication should be administered first to this patient?
recessive pattern. Serum alpha fetoprotein is often elevated in AT. A) Epinephrine.
WHIM (warts, hypogammaglobulinemia, infections, and myelo­ B) Benadryl.
kathexis) syndrome and Chediak-Higashi syndrome are associ­ C) Normal saline intravenous fluid bolus.
ated with neutropenia. Hyper-IgM syndrome is characterized by D) Steroids.
elevated IgM, low IgG and IgA, and decreased T-cell response to
antigens. Affected individuals have recurrent infections similar to Discussion 6-1
other conditions with combined impaired B- and T-cell function. The correct answer is ''A:' Anaphylaxis is a type 1 hypersensitiv­
ity reaction with IgE-mediated mast cell degranulation involv­
Question 5-2 ing multiorgan systems. First-line treatment for anaphylaxis is
What should be avoided in this patient? epinephrine-every time. If the first round of epinephrine is inef­
A) DTaP vaccine. fective at fully reversing symptoms, a second round of epineph­
B) Pneumococcal vaccine. rine should be administered. Dosing for epinephrine is 0.0 1 mgl
C) X-rays. kg intramuscular of a 1 : 1 000 solution. Early administration of epi­
D) Irradiated blood products. nephrine reduces mortality associated with anaphylaxis. Epineph­
rine should be given intramuscularly rather than subcutaneously.
Discussion 5-2
The correct answer is "C:' With a defect in the DNA repair
• Helpful Tip
machinery, ionizing radiation must be avoided as it increases
=� Genera l r u l e of t h u m b for epinephrine dosi n g :
strand breakage and malignancy risk. Live attenuated viral vac­
cines should be deferred in any patient with a suspicion of cellular
i1 1 r · For c h i l d ren < 25 kg, g ive 0. 1 5 mg in a n epinephrine
a utoinjector
immunodeficiency. Irradiated blood products should be given to
• For c h i l d ren and a d u lts > 25 kg, g ive the sta n d a rd
patients with a T-cell immunodeficiency requiring transfusion
dose of 0.3 mg in an epinephrine a utoi njector
to prevent the development of graft-versus-host disease.
CHAPTER 2 • A L L E RG I C A N D I M M U N O LOG I C D I SO R D E R S 31

i s a transient increase i n pulmonary vascular resistance. Sitting


Being the astute ED physician of the day, you decide to admin­ upright has been associated with fatal anaphylaxis due to pulse­
ister epinephrine. After 10 minutes, you note her symptoms less electrical activity cardiac arrest presumably due to inad­
persist and she is now becoming hypotensive. Her father tells equate cardiac filling from severe hypotension with decreased
you that she was recently started on a beta-blocker medica­ venous blood return, therefore; patients with anaphylaxis
tion as a form of migraine prophylaxis. should be placed supine with their legs elevated.

Question 6-2
In addition to establishing an IV, what is the next step in her care? • Helpful Tip
A) Repeat epinephrine. � I n a n a phylaxis, position t h e patient s u p i n e with h i s o r
=.
B) Glucagon. r1 1 r her legs elevated . This w i l l h e l p preserve i ntravascu l a r
C) "Stress-dose" steroids, as the hypotension is secondary to vol u m e a n d venous return t o the heart t o prevent
adrenal insufficiency. em pty ventricle synd rome, which may res u l t i n cardiac
D) Vasopressor infusion. a rrest and sudden death.

Discussion 6-2
The correct answer is "A:' The treatment of choice is epineph­
She improves after two rounds of epinephrine. She is alert,
rine. Beta-blocker medications increase the risk of a more
talkative, and has normal vital signs. Her rash and facial
severe reaction and treatment-resistant anaphylaxis. Between
swelling have resolved. She is admitted to monitor for the
1 5 % and 30% of reactions may require a second dose of epi­
return of signs or symptoms of anaphylaxis.
nephrine. Epinephrine causes both alpha- and beta-adrenergic

Question 6-4
effects, leading to vasoconstriction, tachycardia, bronchial
relaxation, and increased vascular permeability. This effect can
sometimes be blunted in patients taking beta-blockers. If two Which is NOT a sign or symptom of anaphylaxis?
to three rounds of epinephrine are ineffective at fully revers­ A) Periorbital edema.
ing the reaction, glucagon should be administered to patients B) Clinging to parents.
taking a beta-blocker medication. Glucagon has both positive C) Targetoid skin lesions.
inotropic and chronotropic effects on the heart. It does not D) Throat tightness.
involve catecholamines, and therefore is unaffected by beta
blockade. In patients with prolonged hypotension from recal­ Discussion 6-4
citrant anaphylaxis, adrenal hemorrhage and resulting insuf­ The correct answer is "C:' Mast cell and basophil degranulation
ficiency has been described. Consider this when hypotension with the release of histamine and other mediators is respon­
is persistent with an appropriate clinical history. Second-line sible for the local and systemic manifestations of anaphylaxis
therapies sometimes used include antihistamines (H 1 and H) , (immunologic and nonimmunologic) . Anaphylaxis is a clinical
bronchodilators, and corticosteroids. Antihistamines treat diagnosis. Symptoms may onset in minutes to several hours.
itching, angioedema, and hives but are not lifesaving and do Clinicians must remain aware that symptoms may be delayed
not treat airway obstruction or shock. Corticosteroids have as to avoid missing the diagnosis and delaying treatment with
a delayed onset of action and had traditionally been given to epinephrine. Infants cannot describe their symptoms and may
treat protracted anaphylaxis and prevent biphasic reactions exhibit nonspecific behavioral changes such as crying, fussing,
though this has not been supported by the literature. Biphasic clinging to parents, and irritability. (See Table 2-3.)
reactions occur in a small percentage of patients. Symptoms
reoccur typically within 8 to 1 0 hours despite no repeat expo­ After speaking with the girl, you find out she ate cashews
sure to the trigger. 1 hour before she broke out in hives. She eats cashews infre­
quently and thinks her throat may have felt scratchy the last
Question 6-3 time she ate them, but nothing like this has happened before.
Anaphylactic shock is typically characterized as what kind of
shock? Question 6-5
A) Cardiogenic shock. Which is NOT a trigger for anaphylaxis?
B) Neurogenic shock. A) Egg.
C) Hypovolemic-distributive shock. B) Nonsteroidal anti-inflammatory medications.
D) Nephrogenic shock. C) Latex.
D) Mosquito bites.
Discussion 6-3 E) Bee stings.
The correct answer is "C:' Physiologic changes in anaphy­
laxis include vasodilation and loss of intravascular fluid from Discussion 6-5
increased vascular permeability, followed by vasoconstriction The correct answer is "D:' Foods, insect stings (Hymenoptera
and myocardial depression with reduced cardiac output. There insects), and medications-especially antibiotics-are common
32 MCG RAW-H I LL E D U CAT I O N S P E C I A LTY BOARD REVI EW: P E D I ATRICS

TA B L E 2-3 C L I N I C A L S I G N S AND SYM PTO M S OF • Helpful Tip


A N A P HYLAX I S
=� Ana phylaxis with exercise may occ u r u po n eating
Clinical Signs Symptoms r1 1r certa i n foods before exerc i s i n g . Ana phylaxis does not
occ u r if the food is eaten without exerc i s i n g . This is
U rticaria (h ives) Pruritus
ca l led food-dependent, exercise-i n d uced a n a phylaxis.
Flushing Nasal congestion,
r h i norrhea
Ang ioedema (face, l i ps, Itc h i n g of oropha rynx
ton g u e) ( l i ps, th roat, to n g u e)
B ronchospasm (wheezi ng) Dys pnea � QUICKQUIZ
Stridor Co u g h
Hypoxia Vom i t i n g Hypersensitivity reactions to radiocontrast media are due to
Res pi ratory arrest Abdo m i n a l pa i n what mechanism?
A) Immunologic anaphylaxis.
Hypote nsion Dia rrhea
B) Nonimmunologic anaphylaxis.
Tachyca rd i a Pa l p itations C) Secondary to shellfish allergy.
Arrhyt h m i a s Chest pa i n , tig htness
Ca rd i a c arrest Syncope Discussion
Conj u nctiva l i njection Sense of i m pe n d i n g doom
The correct answer is "B:' Immunologic anaphylaxis is an IgE­
mediated systemic reaction to an allergen. Nonimmunologic (for­
Tea ring Fee l i n g a nxious
merly known as anaphylactoid) reactions are non-IgE-mediated.
Contrast allergy is secondary to rapid shifts in osmolality with
causes of IgE-dependent reactions. No trigger may be found fluid changes causing physical degranulation of mast cells and
in some cases. Milk, soy, egg, peanuts, tree nuts (eg, cashews), basophils. Pretreatment with steroids and antihistamines in the
shellfish, fish, and wheat are the primary food triggers. hours leading up to contrast administration can often blunt or
prevent this response. There is no association between contrast
reactions and shellfish/iodine content. Other examples of non­
• Helpful Tip immunologic anaphylaxis (direct mast cell activation) include
=� Foods a re the most common trigger of a n a phylaxis i n exercise, cold exposure, and opioid medications.
r1 1 r c h i l d ren a n d adolescents.
• Helpful Tip
� Tryptase is a s u b g ro u p of serine peptidases fou n d
1 1 1 r in mast cel l s (a n d to a lesser deg ree basop h i l s).

� QUICKQUIZ It is released i nto the serum fol lowi n g mast cell


degra n u lation. Plasma tryptase levels pea k at 60 to
90 m i n utes after onset of a n a phylaxis a n d persist u p to
Which of the following are risk factors for anaphylaxis? 5 h o u rs after the event. Trypta se is often not elevated
A) Age. in food-i n d uced a n a p hylaxis.
B) Gender.
C) Atopy.
D) Geography.
E) Socioeconomic status.
F) All of the above.
On a chilly December afternoon, a 1 2-year-old girl, pres­
Discussion ents to your office with hives and swelling on her face and
The correct answer is "F' Lifetime risk of anaphylaxis is 0.5% to hands. She noted them after walking home from the bus stop.
2%. Atopy is an increased risk for idiopathic, exercise-induced, Alarmed, her mother rushed her to your office. En route, the
food, radiocontrast, and latex anaphylaxis. It is not a risk factor girl had mentioned eating a peanut butter candy on the bus
for anaphylaxis to medications. Anaphylaxis is more common that a friend had given her to try. She has no personal his­
in males until age 1 5 , and in adult females. More cases involve tory of known food allergy. Other than feeling "itchy" from
children. More epinephrine pens are prescribed in the northern the hives, she denies any abdominal pain, nausea, chest tight­
United States than in the southern counterpart. Higher income ness, or feelings of lightheadedness. On further history, it
also seems to be a risk factor. Exercise, acute infection, fever, and is noted that she has experienced hives on and off over the
emotional stress are examples of co-factors that may amplify the past 3 years, mostly in the winter months. She has never had
anaphylactic reaction. issues during the summer. An episode last year prompted
CHAPTER 2 • A L L E RG I C A N D I M M U N O LOG I C D I SO R D E R S 33

epinephrine administration in the emergency department Discussion


when she developed full body hives, wheezing, and hypoten­ The correct answer is "A:' Urticaria are transient, raised, blanch­
sion after participating in the local "polar plunge" into a par­ ing, erythematous, pruritic lesions that come and go over the
tially frozen lake. The family was told to "see an allergist" for course of several hours to days secondary to released mediators
further workup but failed to keep the appointment. ( eg, histamine, leukotriene, prostaglandin, anaphylatoxins such
as C5a, etc) . These mediators lead to changes in vascular per­
Question 7-1 meability. Acute urticaria is common and thought to affect as
What is the best test to confirm your suspected diagnosis? many as 20% of people. The most common cause of acute urti­
A) Ice cube test. caria (hives lasting less than 6 weeks) in children is a recent viral
B) Autologous sweat test. infection. The hives may precede, follow, or occur concurrent to
C) Serum-specific IgE to peanut. the illness, which often leads to confusion about the diagnosis.
D) 24-hour urine histamine collection. Food and drugs are other causes of acute hives. Chronic urti­
E) C1 inhibitor level and functional assessment ( C 1 INH) . caria (lasting more than 6 weeks) is less common, seen in 1 %
of the population. Despite common belief, it is rarely associated
Discussion 7-1 with food. If persistent, chronic hives should prompt workup
The correct answer is "A:' Cold-induced urticaria is a type of for underlying inflammatory conditions such as infection, neo­
physical urticaria associated with rapid-onset pruritus, ery­ plasm, or autoimmunity. Treatment of acute urticaria is with
thema, and swelling after exposure to a cold stimulus. Symptoms oral antihistamines, and in severe cases, a short course of oral
are often maximal after the area is rewarmed. Massive mediator steroids may be warranted. For chronic urticaria management,
release can be found when the cold is immersive, such as swim­ a stepwise approach is used to suppress symptoms with a slow
ming in cold bodies of water, and may manifest with hypoten­ wean looking for recurrence. Second-generation H 1 blockade
sion. The disease can begin at any age. An ice cube test (placing with a nonsedating antihistamines up to four times typical dos­
an ice cube on the arm for 4 minutes, followed by 10 minutes ing may be used. If hives persist, addition of H2 blockade and
of observation) is diagnostic. Treatment is with antihistamines, leukotriene receptor blockade may be necessary. For resistant
with cyproheptadine being preferred in this type of physical cases, immunomodulators (eg, cyclosporine and mycopheno­
urticaria. The problem may resolve spontaneously or persist for late) as well as omalizumab (anti-IgE monoclonal antibody)
several years. Cholinergic urticaria is defined by acute onset of may be appropriate for long-term control.
small, punctate wheals and flares associated with exercise, hot
showers, sweating, and anxiety. It is diagnosed by an autolo­
gous sweat test. Looking for food allergy should be considered
in any patient presenting with acute urticaria and angioedema
but does not explain the history of hive development outside
of this instance. A 24-hour urine histamine collection is help­ You are seeing a 5-year-old girl for her springtime well-child
ful in diagnosis of anaphylaxis or suspected mast cell disorders; check. Mom states her daughter has been doing great overall;
however, it would not be the best choice in this patient. A C 1 however, over the past 1 to 2 years she has noted the child
INH level and functional assessment are used i n the diagno­ seems to have chronic nasal congestion and pruritus. She will
sis of hereditary angioedema. This condition is secondary to a go into "sneezing fits" when she awakes in the morning. Last
defect in bradykinin metabolism leading to recurrent episodes year, she developed hives along her back and legs after rolling
of angioedema typically with an absence of urticaria. in the grass. You examine the child and note pale, boggy nasal
turbinates.

� • Helpful Tip
Physica l u rticaria comes in va rious forms, i n c l u d i n g
Question 8- 1
What would the most effective medicine be in treating her
I I cold i n d uced, heat i n d uced, c h o l i n erg ic, pressu re, symptoms?
vibration, solar, or aquagenic. A ca refu l h i story a n d A) Nasal antihistamines.
c l i n ica l reprod u c i b i l ity a r e d i a g n ostic. B) Antileukotriene therapy.
C) Nasal corticosteroids.
D) Oral antihistamines.

� QUICKQUIZ Discussion 8-1


The correct answer is "C:' Intranasal steroids are the most effec­
tive medication for treatment of allergic rhinitis. Allergic rhinitis
What is the most common cause of acute urticaria in children? is commonly noted in older children (must be older than age 3)
A) Viral illness. with primary symptoms of rhinorrhea, nasal or ocular pruritus
B) Antibiotics. (or both), eyelid edema, and tearing. It is an IgE-mediated disease
C) Pet dander. associated with other atopic conditions. Its lifetime prevalence is
D) Venom hypersensitivity. 25%, with 80% of cases developing in childhood. Allergic rhinitis
34 MCG RAW-H I LL E D U CAT I O N S P E C I A LTY BOARD REVI EW: P E D I ATRICS

occurs frequently in times of high allergic exposure. For chil­


dren with tree and grass pollen allergy, this is commonly in the • Helpful Tip
e.� Consider the eva l uation of a l lergies in c h i l d ren
spring (March through June in the northern hemisphere) . The
fall allergy season is predominantly mold, ragweed, and weeds r1 1 r m a n ifesting with recu rrent s i n u sitis or otitis media.
spanning from mid-August to early frost. Perennial allergens, When a l l ergies have been exc l u d ed, a natomic

such as dust mites and animal dander, may lead to year-round obstructions preventing normal d ra i nage s h o u l d be

symptoms. Allergic rhinitis is uncommon in children younger suspected.

than age 3, therefore; alternative diagnoses should be consid­


ered, including foreign bodies and infectious rhinitis, especially
if occurring in the winter months. If the rhinorrhea and conges­ • Helpful Tip
tion is not associated with pruritus and is triggered by strong e.� Nona l l erg ic r h i n itis with eosinoph i l ia syn d rome (NARES)
odors, changes in temperature, foods, emotions, or changes in r1 1r ca n present s i m i l a r t o a l l ergic r h i n itis, with eosinophils
humidity, it is more likely to be vasomotor rhinitis. This type of noted on nasal smear cytology, however wi l l have
nasal symptom responds well to nasal antihistamine sprays. If negative skin testing and seru m a l l erg en specific lgE. It
symptoms are poorly controlled with intranasal corticosteroids is thought to be loca l ized m ucosa l a l l ergy, thoug h the
an additional agent (options A, B, or D) should be added. mecha nism is poorly u n d e rstood. It is typica lly seen i n
a d u lts a n d is treated with n a s a l steroids.
You continue to treat the girl for her allergies for the next
several years, but her symptoms are becoming more dif­
ficult to adequately control with medications. She was
treated for sinusitis twice in the fall during ragweed season.
Her mom would like to consider starting her on allergen
immunotherapy. A 13-month-old is brought into your office after developing
hives 1 0 minutes after ingestion of scrambled egg yesterday
Question 8-2 morning. This was her second exposure to scrambled eggs.
What is true about allergen immunotherapy? Mom immediately gave her diphenhydramine, and the hives
A) In monosensitized children, allergen immunotherapy can resolved shortly thereafter without progression of symptoms.
reduce the rate of development of new allergen sensitivities. Mom brings her in today and states she wants her "tested for
B) Immunotherapy is protective against the development of everything!"
asthma in patients being treated for rhinitis only.
C) Therapy for 3 to 5 years may provide long-lasting remission Question 9- 1
of symptoms in a third of patients. What is the appropriate test to perform in this patient?
D) Immunotherapy causes several changes to the immune A) Skin prick testing for egg.
system, including induction of T regulatory cells and B) Serum IgE testing for egg.
IgG4 (blocking antibodies) production, as well as down­ C) Intradermal testing for egg to overcome the recent anti­
regulation of allergen-specific IgE. histamine use.
E) All of the above. D) Panel scratch testing for multiple foods, including those
currently in her diet.
Discussion 8-2
The correct answer is "E:' Children or adolescents not improving Discussion 9-1
with medications and allergen avoidance should be referred to a The correct answer is "B:' Skin testing identifies positive
specialist for allergy testing and consideration of immunother­ IgE sensitization and is thought to be the most sensitive test
apy (subcutaneous "allergy shots" or sublingual) . The US Food for detecting allergy. It is measuring released histamine in
and Drug Administration (FDA) recently approved 3 allergy response to tested allergens, and therefore patients must be off
tablets (sublingual therapy), two for grass pollens and one for antihistamine medications prior to testing. A general rule of
ragweed. Immunotherapy involves the repeated exposure to thumb is that long-acting antihistamines (eg, cetirizine, fexof­
escalating doses of a specific allergen. The child or adolescent enadine, and loratadine) must be stopped 5 days prior to test­
must have a documented allergy (history, allergy testing) . The ing, and shorter acting antihistamines (eg, diphenhydramine)
goal is to increase immune tolerance to the antigen. Immuno­ stopped 48 to 72 hours prior to testing. If the response to the
therapy is effective treatment for allergic rhinitis, allergic con­ histamine control is blunted during testing and the patient has
junctivitis, allergic asthma and venom hypersensitivity. Severe been off oral antihistamines, you must review the medication
or poorly controlled asthma is a contraindication to immuno­ list to evaluate whether other classes of medications (eg, anti­
therapy as it may induce bronchospasm. Subcutaneous therapy depressants) are leading to antihistaminergic effects. Serum­
may cause anaphylaxis and should be given in a medical setting. specific IgE testing (formerly known as RAST testing) measures
Sublingual therapy is safer, has fewer allergic reactions, and may the amount of allergen-specific IgE produced and circulated
be given at home after the first dose. Currently, immunotherapy in the serum. A patient may remain on antihistamines for this
is not recommended for food allergies. type of testing. It is helpful when skin prick testing cannot be
Another random document with
no related content on Scribd:
Dirk’s stem huilde van onmacht en drift, maar Kees
verstond niet.

—Wat-te?.… wa’.… tè? krijschte die in strek-stand van


heel z’n lijf, tegen den zij-wand òpgeschuurd. Heet-
koortsige vloeiingen van vermoeiing voelde ie door z’n
armen trillen. Hij kòn niet meer; toch hield ie nog in
radelooze spanning vast, hoog z’n vork. Dirk
schreeuwde weer. Maar vaag hoorde Kees
stemmeklank in windstoei, zonder woorden. Nou ging
’t niet langer.—Dàn moest de vracht maar smakken.
Z’n borst blaasbalgde, z’n beenen gloeiden en trilden
méé, en in heeten schater woei de wind hem
hooifladders in de zweet-dampende tronie.

—Veùr op.… Veùr òp! krijschte Dirk, doldriftig en


zweetrood woelend met z’n armen in den gouden
hooibrand, woestnijdig dat Kees ’m niet verstond.—In
spanning die hem dreigde te doen flauwvallen hield
Kees nog òp de vracht, en vaag nu hoorde ie de
woorden van Dirk achter windgerucht, begreep ie op
voorkrat te moeten aanzwenken.—’t Was alsof ie bij
[214]’t begrijpen, nieuwen krachtstroom in zich voelde
gloeien.

Zacht weer schuurde ie z’n vork langs den berg.—De


wind joedel-doedelde weer, dat ’r slierten als
klapwiekende goudvleugelige vogels hem
omzwermden. Z’n oogen brandden en piekten,
morsten vol heet zand.—Maar voort ging Kees tot ie
Dirk èven zag bij ’t voorkrat, weggedoken in ’t hooi, die
eindelijk met wijdopen armen, z’n vork leegrukte.—
Zonnedaverend vlamblondde de wagen in al hoogeren
uitgroei, en stilpeinzend paard bleef turen in
droomgeduld, kop naar den grond, waar ’t hooi om z’n
stille pooten warrelde en walste een jolige wind-
horlepijp. De dissel, krom als ’n rooie bazuin, vuur-
gloeide tegen ’t hooigoud in, en feller blinkerde z’n hel-
koperen punt als ’n zonnige glanzige flakker in de
lucht. Op enkele hel-roode spaken zonneschramden
lichtkartels, maar heel ’t wielas-spul dofte donker-rood,
in schaduwkoel van hooikar-heupen. Hevig in blauwen
brand, straalde de hemel, boven ’t groen. En hooger,
de blonde rug van het gevaarte zwol áán in ’t
zingende licht. Dirk er in, er op, pal in ’t broeiende
hart, woelend in wankel met z’n stronkige ruigheid van
lijf onder de violet-blauwe kiel, z’n brons rooie, ramp-
angstig gemartelden kop, hoog tegen de lucht.—Als ’n
titan worstel-tastte en bouwde ie bijeen, bekookt,
begoten van zonnevuur; licht dat schroeide z’n
vleesch, groef en dreunde, sloeg en vrat in ’t hooi;
hooi dat in sprongen veerde, ònder handen- en
voetenwoel van den bronzigen titan. Z’n geel haar
leek brandend hooi en z’n strooien hoed droop zweet
af, op z’n heetrooien brons-zonnigen nek.—Telkens, in
verhitte prikkeling viel ie neer op de vork van Kees, als
ie de punt èven zag opschuren boven ’t voorkrat. Maar
plots dan weer, patste windruk ’m scheef, vloekte en
verwaggelde ie hulpeloos terug in ’t hooi, in driftgreep
z’n knuisten de oogen inwrijvend. Met heeten
windraas was hooizaadje z’n oogen ingewaaid als ’n
gloeiende vonk. ’t Brandde pal op z’n appel en z’n
morsige knuisten wroetten en woelden langs z’n
priemfel stekend oog. Het gloeide ’r als prik van heete
speldpunt. In razende pijndrift smakte ie zich neer,
probeerde [215]met nijdig gewrijf ’t zaadje er uit te
persen, dat ie ging voelen, groot, als ’n brandende bol
onder z’n oogleden.—’t Bloed gloeide ’m onder de
nagels. Hij blies, pufte en vloekte al driftiger. De zon
zengde z’n kop en door de hitte die ’m roosterde,
branderig verklefferde, dartelde en joedelfloot de wind
’n sarrig deuntje, bij vlagen, hoog en laag. Dirk dreigde
te bezwijmen van woede, pijngepriem en roosterende
ovenbenauwing. Onder hevig gewrijf wanhopend,
begon z’n oog te schreien, voelde ie ’t priem-vurig
stekige verzwemmen in traannat, alsof ie ’t zaadje in
’n hoek gedrongen had.—Hij kon weer ’n beetje kijken.
’t Brandde er wel, en vòòr zich zag ie niets anders dan
groene en rooie flakkering scheem’ren. Hij speurde
wel geen hooi, geen lucht, geen vork opsteek, maar hij
kéék toch. Na ’n poosje zag ie Kees uit ’n flakkerig-
groenigen krans òpduiken, en den Ouë verderop. Nou
maar weer voort, in worstel, tegen hitte en windsar.…
En kalmer als ie kon, bedaarder vooral! Met open
armen ving ie de hooigolven op, stond ie klein-
weggezakt, en half-lam gebeukt van vermoeiing te
puffen naar adem, in de rollen, stootte ie de
opgevangen stapels vooruit met z’n kin, z’n borst,
breed begraven in den hooi-blonden brand.—Terwijl
z’n achterlijf naar achteren woelde persten z’n
handen, zwaar driftig bijéén.

—Hee Ouë! kaik rais achteran? hoe stoàn die? Nai


Kees!.. jai nie. Staik jai moar deur!.. Hee!.. Ouë!..

Gerrit kwam aanklomperen, klos-klos, knerpend op ’t


grint. Z’n kop stond wrang van stuurschheid.—Hij
voelde zich geroosterd, hij zuchtte.. dat ie vast
hooikoorts had.. dat ie d’r niet meer tegen kòn. Met z’n
hark geschouderd, keek ie òp naar den wagen.
Tusschen z’n beenen sliertte hooi, hollend langs den
dijk.

—Hoho! daa’s ’n meroakel-wind! die sloàn je boel


òm!.. je hangt d’r hier àn dut sai heuldegoar òfer!

Dirk raasde van wagenberg, keek angstig naar Ouë’s


mond, om beter te begrijpen wat ie zei.

—Hang’k òfer?.… die pestwind! je ken dur.… f’rdomd


soo niet loaie.… daa’s gain doen.… da’ duurt d’r puur
twai [216]uur langerst.… iedere.… woàge.… en.. en..
as je dur dan nòg thuis kòmp!.. Strak en an lait.… de
boel veur de waireld.

Ouë Gerrit, met half dichtgeknepen oogen achter den


wagen, één beverige hand, als afdakje op z’n voorkop
geluikt, oploerend tegen ’t schelle zonnevuur, naar
gouden schaduwrug van gevaarte, schudde z’n
lokkenkop, dat z’n krullen flapperden en z’n zilveren
baard danste heen en weer, op z’n borst.

—Hoho! hou jai d’r moar stiekem deuse kant hee! dan
komp ’r t’ met juustighait in!

—D’r is nie te loaie, veur de tuikigste kerel nie! aas je


de wind dwars in je woàge hep! he’k ’t nie veuruit sait!
Kaik sàin doar! op ’t land.… kop van ’t poard teuge de
wind in, daa’s smeert. Moar goan d’r nou op de daike
’rais dwars stoan?.… debies! debies! is main ’n
kattebakkie.
—Nou, schreeuwde al zwakker stem van ouë Gerrit,
met angstigen kijk heel de polderlucht doorzoekend,—
hou je d’r moar goed, deuse kant uit.… vier en vaife
nie genog.… dan wâ sachies ansjouwe.… dan komp-
ie d’r wel.—

—Joa! makkelik sait.… jai stoan d’r hier nie laifendig


te f’rbrande.… da je de boel onders je almenak
wegwoait.. Main kristis! wâ da’ hooi hiet kookt!.. je
brant je achterlap d’r an! ’t stoan t’met te broeie
onders je poote.… hier Kees! hier!.. joa! ikke ken d’r
niks an doèn.… nou wair àchterop!.. ikke stoan hier
puur op de valreep! kaik d’r selfers moar.… t’met lait ie
veur de waireld! alles is hier.. wegsakt.. f’rfloekt aa’s je
loaje ken.… en je poote stoan aas in hait lood!.…

Weer schoof Kees, in arm-hoogen strek z’n vork van


voorkrat naar achter, in hevigen spierzwel en stronkige
krachtrukken van z’n lijf. Schurend en hijgend draaide
ie op achterkrat áán, naar Dirk toe, die te wachten zat.
Met opengesperden mond, strakwije lobbes-oogen,
ingebukt op z’n knieën, over z’n hooi heèn, rukte hij de
vracht naar zich toe, plompte ie neer, wijd-armig op
den brand-gouden golf, in woest gewoel en krakerigen
druisch van de rollen, waarin ie half verzoop en weer
overeind spartelde met woest armen- en beenen-
verweer.—Z’n kop gloeide en glom [217]als was ie met
boter ingesmeerd; z’n geel haar droop zweetvet af in
z’n nek, die dronkenrood zwol en branderig pijnde.
Beving van martelende, gehinderde werkdrift zat er in
z’n hoekige kaken en beving in z’n handen die persten
en duwden, rolden en smakten. Even moest ie, half
zittend, uithijgen. De wind bleef joedelen rukkerig om
z’n hoofd, stootte zwoelheete luchtkolken over ’t
wagen-gevaarte. Ingekneld en verbroeid, met armen
en beenen begraven onder ’t hooi, voelde ie ’n loome
bezwijming in ’m zakken, ’n heete koortsig-kruipende
verlamming door z’n lijf. Zoete hooigeur en
bloemdroesem stroomde méé met zwoele windvlagen,
z’n hijgenden mond in.—Zonnevuur blakerde z’n
hersens, z’n schouders, z’n rug. Hij voelde zich als
aan ’n spit gestoken, roosterend boven vlammenlik.—

Heel de gloei van fellen zomerbrand ziedde zich uit op


den gouden bergwagen. De hemel, wijd-hevig blauw,
dampte trilhitte uit. En tastend, vloekend, onrustig, en
sarrend-speelsch in de snikwarmte, joeg de wind, floot
ie z’n vlaagdeuntjes op doedelzak en spothoorns,
dartelend en grillig; blies ie als ademhaal van ’n
nijlpaard, pufte dan weer aan, kucherig en zwak, als
hoest van ’n grijsaard, schaterde plots weer òp als
wild stemmenkoor van jubelende cherubijntjes,
meesliertend heeten stroom van landgeuren uit klaver-
en groentevelden, tusschen brandenden warrel van
stof.—

Al loomer voelde Dirk bezwijming opkruipen in z’n lijf.


Z’n handen beefden, z’n oogen loken. ’n Zachte neig,
duizelig en slaperig-bedwelmend, omkronkelde ’m,
soezelde en trok ’m in ’t hooi. Hij voelde dat ie zich
moest laten smakken tusschen de brandende rollen,
in den zwijmel van snikhitte, windmartel en zwoel-
geuren rondom.—Maar weer ’n heftige windvlaag
schoot pal in z’n gezicht, frischte ’m èven òp,
vergloeide erger daarna weer z’n oogen en hals, in
blazende heete tochtvlagen.—
Zwaarder, lam-loomer waggelde ie voort op ’t
zengende hooi, strompelig en doodop.

Klein leek ’t paardje onder hooigevaarte. Stil-suf in de


hitte, stond ’t mijmerend aan ’t voorkrat, met zachten
fladder van [218]manen. Klein en weggedoken de kar,
met z’n binten en donkergroene zijwanden, in huif van
schaduw gevangen. En donkerder ’t assenrood en
wielspul duisterde onder den berg boven ’t raam en
blond gevaarte, uitgebouwd in welving, ontzaglijk
tegen luchtblauw, als ’n duin van goud, blond-gloeiend
goud, schroeiend in zonnevuur.

Kees had weer ’n zware vork opgestoken, kwam


aanzetten, en ingedrukt op ’t hooi lag Dirk’s kop, z’n
romp en beenen wèggewoeld. Als afgehouen levende
tronie op ’t brandhooi, loerde z’n zorg-gekerfd hoofd
daar, rood-bezweet en grimmig-bekommerd,
doorgroefd van ploeterangst, naar den stapel die
òppieken kwam.—Zwaar in schuur, teende Kees z’n
lijf òp, smeet ie heet de brandende golf in wrang-rooie
tronie van Dirk, die plots z’n armen uit ’t hooi opsloeg
en ademsnikkend, in blaas en puf, de rollen naar zich
toe trok. Schreeuwend achter de ingeknelde stapel
angstigde z’n stem nog:

—Kaik.… jai d’rais Kees!.. hoeke kant ikke uit mot!..


Aifig f’rvloekt aa’s ’k ’t weut.

Kees had z’n vork teruggehaald, stond zelf nog half te


duizelen van pijltjes die z’n oogen, hals, mond en
ooren ingewaaid priemden. Stom zweet-dampend en
hijgend liep ie naar achter.
—F’rfloekt! hai hangt d’r heuldegoar noar deuse kant!..
glad-en-al mis! Strak-en àn is tie d’r weust.… je mot
aêre kant houê!

Dirk schrok hevig.

—Nou.… ikke ken d’r nie sien! barstte ie uit, je stoan


d’r hier te broeie en de wind sloàn je aige blind.… die
laileke kaketoe! woar da haine mot! die gaift.… je de
hande vol! da’ kreng! da’ false loeder.

—Aa’s je nou moàr op ’t achterkrat anstuurt!

—Gaif d’r dan snóf’rjenne niks meer van veure àn!.…


aêrs trap ik doar de boel weer omkant!

In joedelenden sar bleef windruk tarten, de


zwoegbeestige kerels rammeiend. Dirk’s gezicht
spande, en angstigde al meer, bij elken nieuwen,
hoogeren aansjouw van Kees. Hij voelde den wagen
al smakken.—Kees, geen stom woord klagend, stond
[219]sterker te hijgen voor z’n hooibergjes, zich voelend
als ’n geslagen kind, òp en verlamd door windweer,
woest op ’n ding dat ie niet grijpen, niet ranselen kon
in z’n drift. Iedere inboor van z’n vork viel ’m tien maal
zwaarder dan bij ’t begin. Bij elken opzwaai zwol z’n
lichaam tot barstende kracht. Rond z’n beenen
schuifelde een warrel van leven en de zon zengde z’n
lijf, alsof ’t op ’n brandstapel vlamomlekt schroeide.

Witte hitte trilde over den polder, paarsig-ijl tegen


forthoogten, alsof de heele aardkom verteerde in één
schroei van stilwerkend wegvretend vuur. Onder den
dijk gloeiden vèraf, de hel-roode, jubel-roode stolpjes,
vlammen in luchtkobalt uitgebarsten dóór ’t porcelein-
gloeiend azuur.—Hooiwagens blondden in goudbrand
en schelven dampten witte hitte uit. Telkens, als heete
blaas-adem, vlaagde de wind òp, zwoel tegen
werkers-zweetkoppen; kittelkilde even in d’r ontbloote
klefferige halzen. Hun kleeren schroeiden ze op ’t lijf.
Hun handen en polsen gloeiden, oogen en hersens
doften. In den ban van hun martelzwoeg, tusschen
wind en snikhitte, ging één stille onuitgesproken
begeerte naar koel nat, ’n snak naar ademhaal, ’n
verlangen naar rust in schaduw-gras, om èven maar
lenden te rekken, lijf te woelen in luien genotstroom
van leven. Zacht beslopen voelden ze zich in loomen
zwijmel. En sterker blakerde zon, plakte en zoog hun
goed vast aan de brandrige leden.—Kees, ongewoon
te hooien, voelde al warmere koortsrilling in z’n
polsen, en jeuk in ’t bloed. Z’n gemartelde keel snakte
naar kou.—Dorst droogde heet en jeukerig z’n
gehemelte uit.—Z’n mond stond dor, in droge
branderigheid. Dorst vampyrde op de warme stik-
benauwde werkers-strotten, als ’n dier dat tergde, hun
keel in brand kon zetten; ze martelden tot ze vloekten
en raasden van gift en onmacht tegen de zon, die
blakerfel, z’n àl witter vuur neergloeide op hun
koppen; de zon, de groote, geweldige schroeier, die
hun oogen doorpriemde en hun half-naakte, zweet
dampende zwoeglijven zengde, vermartelde zonder
genade.—

Zoo, onder worstel tegen wind, die de mannen


grommen deed [220]en huil-vloeken, groeide de wagen
hooger òp, als ’n berg, ontzaglijk van goud-hel geweld
in de peillooze luchtzee, eindloos gloeiend blauw, en
de hooge woel van Dirk op de kar daar, tègen ’t
hemelhelle licht in. Al langzamer, in waggelgang kroop
en worstelde ie z’n lijf door ’t hooi, telkens met z’n kin
op ’t brandende gegolf, z’n armen verspartelend in de
rollen. Achter elkaar, holderdebolder, in sliertenden
kringdans en grissenden zwier, fladderden hooihalmen
de lucht in, over dijk en wei. Verder op, Ouë Gerrit
strompelde nà, wàt ie vatten kon, met z’n hark.—

—F’rdomme, wa’ da’ hooi brant.… aa’s ’n vlam.


Debies! schreeuwde Dirk naar beneden van ’t
geweldige gevaarte, blauw verwapperend z’n kiel op ’t
goud. De kin ingeperst op ’t hooi, zag Kees alleen
Dirk’s kop, angstig-wachtend op nieuwe vork.

—Hoeveul.… hai je d’r nog Kees!

—Nog ’n vaif vorke, galmde ie terug, met z’n rug naar


Dirk gekeerd. Bevend van zwaarte had ie z’n vork
òpgezwierd, en ruk-hevig boven z’n kop gesmakt. Als
loerde de wind plots naar z’n zwaai, zòò joedelde hij
op Kees àf, plukte heele bossen uit z’n vork weg, en
regende woeste gouden warrelpijlen om z’n zweet-
tronie en lijf.

—Effe hooger je vork Kees! ikke kèn d’r soo nie bai!

Kees waggelde voort, in den brandenden fladder en


sliert van hooi. Z’n adem snikte, brak àf in de keel.…
Hij voelde weer dat ie niet hooger kòn. En Dirk zat te
vèr, durfde niet trappen, uit angst dat de boel zou
inzinken. Verlegen, klokkerde ie ’n slok in uit z’n
veldflesch, die gekookt gloeide op ’t hooi, en loerde,
wachtte. ’n Minuut in worstel tegen wind, bleef Kees
staan op z’n teenen, blindgegooid van licht en
hooistrooisel, vloekend rauw, uitzinnig van drift, dat
Dirk niet greep. Maar die wachtte tot Kees hóóger op
zou steken. Woest, in fladder zag ie hooi verslierten
zonder dat de vork naar ’m toeschoof. Dirk bòven
vloekte bezeten, Kees beneden, stond te gloeien van
gift, adem-òp, dat Dirk niet vast greep.—Z’n kop stikte
in hooipijlen en stofdotten; z’n ooren, haar en oogen
werden [221]volgestort van brandend gefladder.
Brullend schreeuwde Kees:

—Nou Dirrèk!.… hep je sain nie an achterkràt?!

—Vàst nie! kom d’r hier op sai, f’rdomd aa’sk sain


hoale ken.… die glad.. joànes.. van ’n wind.

Hij had geprobeerd Dirk, maar waggel-zwaar was ie


achterover gesmakt, z’n kop pal in den fellen zongloei.
Nou kon ie geen woord meer zeggen. Zacht schoof
Kees òm, van achterkrat naar zijhoek. Windgesar floot
om ’m heen, raasde z’n oogen blind, z’n kop duizelig
van hevige inspanning en martel. Woester
schreeuwden ze tegen elkaar; cyklopischer in
weerstand en krachtzwel van spieren en lijf
aangespannen. En toch was ’r ’n angstig elkaar
zoeken te begrijpen met één woord, ’n schreeuw van
rauwen vloek, gegooid boven op ’n woedekreet van
wanhoop en vergeefschen ploeter.

Ouë Gerrit had ’t afgewaaide te hoop geharkt en


vastgeboord aan z’n vork. Stil, puffend in ademhijg
loerde ie schuwtjes òp naar den wagen.
—Hoho! da’ pakt skaif uit, pakt skaif uit.… aa’s die d’r
goed-enne-wel an de dorsch komp hiet ikke-en-gain
Gerrit meer!

Dirk raasde wat van den bergwagen af naar den Ouë.

—De leste vork goan d’r alewel òp.… en dan sel ’t


pondere sain.… hoe stoan die?

—Nou t’met heuldegoar f’rsakt op de windsai! hu!


hu!… kraaide de Ouë schel-scherp terug, handdakje
beverig voor z’n oogen gekneld.—

—Leste vork en dan pondere alewel!

Hijgzuchtend woelde Dirk op den berg, in laatsten


rolslag. Wind floot snelleren joedel en spotte uit blauw
luchtewelf, in sliert en slag, flap en dans.

—F’rdomd aa’s ’k ooit soo loaie hep! schreeuwde Dirk,


’t hooi is d’r wild aa’s ’n knain.… ’t springt d’r onders je
poote wèg. Hee, Kees!.… wa doèn jai?.… sloap ie?.…
hooikoors?.…

Kees was op den dijk tegen de laatste hooischelf


ingezakt, met z’n brandenden rug op ’t gloeibergje. Hij
voelde koortsvloeiing in z’n bloed, ’n matte doffe
gloeiing in z’n handen, ’n [222]piekenden brand van
prikjes in z’n polsen. Z’n oogen holden verschroeid in
z’n kop.

Nou moest ie, al was ’t maar tien minuutjes even


uitblazen, met oogen toch star open, want anders,
voelde ie stellig, zou hem slaap bekruipen, matte,
paffe luier en zwijm..—
Hoog en geweldig uitgebouwd reuzigde de wagen als
goud duin-gevaarte tegen ’t brand-blauw van den
hemel, peilloos trillend van hellen zonneblaker; licht,
dat in heeten windwarrel langs weien en bouwgrond
één kleurbeving goot, brio van heete glansen,
zomerhel en zònnegloeiend over riet, koorn en
groenten vertrilde.—Dirk zat op z’n knieën in ’t hooi te
puffen, wachtend op Kees, blij dat ie ook even kon
uitademen. En strak in stommen staar, keek ie hoog
van z’n goudberg eindloos polderland over.—

Zijïg-zachte warmglanzende goudgolving moireerde


door ’t hooi-zwad, en de nog groen-blauwe
koornhalmen ruischten en zuchtten als orgelregisters
waar lucht-ziel in áánvloeit. Trillend gloeiende lucht,
laaiend blink-goud van zonnigen zomerbrand vrat in
op dakjes, karrenrood en groen, en witte wasemhette
nevelde neer, tusschen gloei-gouden schelven en
schoovensleur van karwei-hokkers. Windwiegel
verstoof gouddamp bij iederen stap der landwerkers.

In de heet-zonnige lucht, boven eindloos-vlammend


goudgroen van weiland, zoemden en klank-snorden
goud-vliegjes en wespen. Boven de donker-
groenende klavervelden, die dampten in gouden hitte,
spartelde ’t in de geurende zonnelucht van àl diertjes.
Hommels in d’r kleurig fluweel schommelden en
gonsden en verzongen met hun tril-vleugeltjes,
zomerlied van honinghaal; plots wègzwaaiend op
windstoot uit hun vaart-vlucht naar de wei-bloemen.
Wespjes in d’r spitsige pracht, zwevend en trillend op
één plek, in hun goudvleugelige vlucht bepoeierd met
zonnestof, zwierven hoog en laag in kransjes van licht.
Gewemel en gezoem overal,—tusschen purper
smeulende en hel-paarse klaver en gras,—van
gloeiende torretjes, in fonkelgoud en weerlicht-blauw,
vol gloedjes en glansjes dooreenzwermend; kevertjes
rood, in zonnegloor, als heete lak-vlammige
[223]kruipertjes, rondstoeiend om spinnewebben, die
als lichtend rag zij-ïg elektriseerden en vonkten in den
zonnedag.

In geurenzwijmel dronken, volgesapt van zoeten


honing, slingerden de hommels door ’t luchtblauw, en
de klavervelden in den rose-witten en rooden
bloemengloei, knapperden van licht. Op dwarspaadjes
en dijken in wilden wriemel van madelief en
boterbloemen, die dropen van glans, rookte ’t walstroo
rond, gelige trosjes, uitademend zaligen zomergeur,
diep bedwelmend; zoeten geur van zonnegrond, als
vijgenreuk rondrookend heel den polder door, in
gelukaangeurenden juich van levenslust. ’t Heete gras
gloeide, hoog prachtgroen, heet goud-groen in hellen
glinster, rag en lichtend in ’t weidevuur.

De polderplasjes wemelden van leven, zogen


hemelblauw in, warmdonker terugspiegelend op ’t
watervlak, en tusschen het ruischriet, snerpte gors,
zongen er schelle vogelkeeltjes òp, van kievit en
spreeuwen, floot lokkend-klaar eenzame wielewaal.
Zonnige schitter plaste op de watertjes, die dampig en
fonkelspattend, kronkelig en zonnewiegelend,
tusschen ’t glanshelle weigroen slingerden.
Koeiendrom op ’t malsch gesapte weiland, waar et-
groen laag gloeide, glansde en blankte, in git en wit, in
rood en blond, grazend doom’rig en traag-rustig op ’t
afgemaaide gras, dat sapte en bruiste van gloed-licht.
—Soms plots in draf, holde ’t vee langs sloot-greppel,
spròng zot en onbeholpen terug in dwaze gebaren.
Soms, op lui-bijééngegroepte koeien, zonnebroeiend,
zakte parmantig ’n spreeuwpaar, stappend op hun
ruggen als zwart-glanzende wandelaartjes, happend
vliegen, die om neus en oogen van de koeien sarden,
in kwellenden kringetjesgons.—Paardjes en veulens,
roodglanzend, schimmelhitjes, grijzigblank, holden
door ’t zomergroen, in slanke vlucht van manen en
staart. En telkens van de dammen, waggelden òp de
hooiwagens, hoog tegen trillend azuur, hel-blond,
goudgeel met den woel der blauw-bekielde kerels er
òp, als uitgesmeed tegen ’t hemel-ontzaglijke; de
wagens achter elkaar in krakenden hort, in piependen
kreun en zuchtenden steun, den bestráten polderweg
àf, plots wiel-ratelend, omkolkt van zandhoos. En
hoog in de lucht, dwars [224]door donkeren klapwiek
van kraaien waaierde blanke vlucht van leeuwrik, z’n
zilver gerucht verjubelend door zonneruimte.

Overal zomervuur drupte brandende bloem-boeketten


van zengend goud op ’t heete gras.

Heel de polder lag droomrig stil in den zonneblaker te


roosteren. Alleen van den blinkenden bekeiden
hoofdweg warrelde stof òp bij windrukken, in zwier en
hoos, van damp en licht. Tusschen ’t gras brandde ’t
bloemgezwerm, geel en blank, rood en brons, en hel-
vlammige klaprozen bij dijken, slankten uit boven ’t
groen, in fel kalkoenerood. Zomerhitte snikkend lag te
zwijmelen tusschen karwijschooven en hooi. Broei-
wind goot wilde striemsels van kleurbeving over ’t
land, telkens anders verglijend van glans. En de
oneindige polderkom, lag als ’n luidlooze wereld van
stil gebeuren, brandend en murmelend, zangerig
wind-doedelend en doorplechtigd van ruischende
stilte-muziek der weien,—matelooze graszee en
luchtewelf.—

Op dijken, achter stolpjes, sjokkerden soms kindertjes


áán, roodharige en blonde jochies, speelsch zwierend
vóór boerderijtjes, omblakerd van zonnegloei. En
soms doken òp, uit erfjes, zoo van vèr, meisjes
roodrokkig, heel van vèr, als dribbelende
kersenmandjes in zon, vol bloeiend rood en fleurigen
schater van kleur. Inéén doken ze weg achter
kippenloop of stolp, en stil-leeg dan weer stonden de
dijken heet te trillen, groen, tegen ’t hemellicht;
verdwenen al meer schelven op de wagens,
vervlakkend ’t land uit z’n bergjes-onrust. En
rondomme, uit luchtvuur en gras, vlindergestoei
zwermde áán, witjes met blanke zeiltjes doorzond, in
argeloozen klapwiek van vleugeltjes, boven klaver en
walstroo, distels en riet. Ze fladderzwierden zeilend
door den blaker-gouden, wit-gouden dag. Goudvosjes
en dagpauwoogen in glans brandend, verterend in
eigen gloed, zwermden mee, méé in zomerzwijm,
dronken en half-verstikt al in zoete walm van
bloemengeur waarin ze vlerkten.

Dirk zat nog gehurkt uit te rusten op z’n kar, in


slaapsuffen dommel. [225]

Ouë Gerrit, stug en chagrijnigend over iets dat ie niet


zeggen wou, harkte weer bij aan den overhellenden
wagenkant. Met z’n handen rukte ie zware dotten
langs de wielen af, gooide ze neer op den dijk, waar
ze ritselden en zacht kropen om z’n voeten, tot ze
plots, met ’n fellen windflap mee de lucht instoeiden,
heel vèr.

Kees was uit z’n koortsige loomheid en dommel wat


opgefrischt. Dirk had ie ’t achterbindtouw opgegooid,
die in angstigen hurkkruip op ’t hellende gevaarte bij
voorkrat ’n knoop sloeg. In ’n zwiep sliertte hij ’t touw
weer naar beneden, om te sjorren. Met één been
tegen achterkrat aangeperst, z’n voet in ’t gloeihooi
wroetend, heesch Kees zich òp tegen ’t touw en
sjorde den ponder. Plots schreeuwde Dirk kort:

—Hohai! feurbint is uit de kaip!

Dirk nijdig en vloekend met wrangen kop, woelde weer


langs den ponder òp naar ’t voorkrat en haakte
bindtouw in de pondergleuf.

—Arrait! schreeuwde ie driftig.

Kees, weer met één voet in ’t hooi gestampt om zich


af te zetten van den grond, begon opnieuw met twee
handen in herkuulrukken te sjorren, maar weer van de
hoogte klonk wanhoopstem van Dirk.

—Hohai! feurbint is uit de kaip! f’rfloekte gladjanes!

Woest was ie weer naar voorkrat gekropen, duizend


maal „debies”! razend, met z’n vuist ’t brandende hooi
inwoelend, òp van ongeduld en voortvretende drift.
Weer had ie ’t touw ingeduwd!
—Hoal an!

Kees sjorde weer. Dirk, op den berg, keek in grimmige


spanning. ’t Gevaarte kraakte, kromp en steunde
onder de rukken van Kees, geholpen nog ’n beverig
handje, door Ouë Gerrit.

—Hoal an! hai hou dur! hoàl.…

Bons! ’n ruk-scheur door Kees’ arm, en weer was ’t


touw uit de pondergleuf geschoten.

—Hohai! d’ruit! da’ laileke kreng! die pestneger!


krijschte stikkend van drift Dirk weer. [226]

Dirk, in zwoeg, bloedrood nu van zorg, angst en


nijdigheid stond overeind op den wagen, stapte
staande naar ’t voorkrat.

—F’rek aa’s ’k dâ snap! dàn is de feurbint te kort.…


debies! hai ken d’r nooit nie komme soodurwais! jai
mot d’r van achtert te vèr of’rhoàle.…. dan spant hài
d’r hier te veul.… van feure.… enne springt d’ruit.… de
boel lait.… te wàit hier.… dâ hai je dur nou van mit die
staive wind! f’rvloekt.. alle duufels!.… da’ hooi brant je
je laif àf.…

In wanhoop greep Dirk z’n hoed en smakte ’m op den


dijk, dat de zon nu z’n naakten gelen kop omkookte.—

Rauw-driftig geschreeuw klonk er òp en weer, van en


naar den wagen, de stemmen verwaaid in
windgerucht.
—Ken ie van ònders nog rekke.. schreeuwde Dirk.…
hai lait.. weer..! Nou Kees.. Ouë!.. sachies àn! hoal!!

Weer sjorden de kerels achter ’t krat, in dampenden


lijfzwoeg, borsten blaasbalgend in hijg. Weer zakte
gevaarte zoetjes, daalde Dirk méé, angstig, gejaagd,
z’n zweetrooie tronie gekeerd naar den ponderkeep.
Maar weer bij sterke spanning schoot ’t touw uit,
stopten de kerels, en rees in zuchtenden ademhaal,
de hooiberg weer òp.—Dirk z’n kop bloot, woest, half
dol van wanhoop, vloekte als ’n gek, bulderde keek òp
naar de zon die z’n oogen blindsloeg van licht, stond
half te stikken van onmacht.—

—Ik bin kepot, kepot! kreunde Dirk met gesmoorde


stikkende en stervende stem.—

—Aa’s je d’rais ’n stuk touw knaipt an ’t feurbint,


schreeuwde zwakkelijk Ouë Gerrit, de kaip is d’r.…
afslaite.… hu! hu!

—F’rdomd.… daa’s net! gromde Dirk even òplevend


uit z’n stervende moeheid.—Hee Kees hier!.. bai ’t
feurkrat sit d’r ’n dot touw,.. kaik jài d’r us!

Maar Kees vloekte en wreef, in pijnkramp van z’n lijf,


z’n oogen uit.

—Steek de moord, da’ goan main ’t zweet de lampies


in! snof’rjenne wa’ bait dâ!

Als was z’n oog met ’n zuur gif volgedrupt zoo vrat en
brandde [227]’t kopzweet hem ’t oogenwit in.—Rood
geflakker vlamde ’r voòr ’m. Toch strompelde ie, half
blind naar ’t voorkrat, met Ouë Gerrit achter ’m aan.
Uit ’n hoek grabbelde Kees ’n end touw op. Met
nijdigen ruk sjorde ie ’t voorbint los en kronkelknoopte
’r ’n stuk aan vast. Op één oog voelde ie zich blind, en
pijn schroeide er in, als was ’t z’n kop uitgerukt. Toch
vond ie ’t malligheid te jammeren.

—Kloar!

Dirk op den wagen, trok áán, probeerde of ’t touw lang


genoeg was, niet meer uit de gleuf kòn.

—Te lang, slappies! knoop d’r ’n halfe slag bai! sóó,


nou! nou goan die d’r wel! snó’f’rjenne.… aa’s da
loeder d’r nou nie in blaift.… sprink sain vast op s’n
strot!

Met ’n zware poef smakte Dirk zich neer op den


ponder bij ’t katrol.

Kees en Ouë Gerrit aan achterkant, sjorden, rukten en


strak-knarsend en kreunerig bleef ’t touw kermen in de
gleuf.

—Hoal d’ran manne! hoal d’ran!

Zwaarder perste Dirk z’n lijf op den ponder, z’n


beenen spartelend in ’t hooi, z’n armen krampachtig
om den balk geklemd. Heviger sjorde Kees, dat de
wagen wiegelde, kreunde en kraak-zuchtte en ’t
gouden licht op den hooibrand meezwiepte, op-en
neer, in ’t hemelblauw.

Langzaam zakte Dirk langs pondertouw àf, dat Kees


door ’t katrol geheschen had. Met hun drieën nu

You might also like